[ /tv/ /rf/ /vg/ /a/ /b/ /u/ /bo/ /fur/ /to/ /dt/ /cp/ /oe/ /bg/ /ve/ /r/ /mad/ /d/ /mu/ /cr/ /di/ /sw/ /hr/ /wh/ /lor/ /s/ /hau/ /slow/ /gf/ /vn/ /w/ /ma/ /azu/ /wn/ ] [ Main | Settings | Bookmarks | Music Player ]

No.77310 Reply
File: 134322681004632.jpg
Jpg, 73.23 KB, 496×480 - Click the image to expand
edit Find source with google Find source with iqdb
134322681004632.jpg
Новый математический тред.

ВНИМАНИЕ, АНОНИМУС
Если тебе нужна подборка книг по какому-то определенному разделу математики, или же ты не знаешь, с чего начать, то не поленись, зайди сперва сюда, а уже потом задавай свои вопросы.
http://lj.rossia.org/community/studium/1717.html#cutid1
  
Полезные ссылки:
Калькулятор и редактор формул: http://ru.numberempire.com/texequationeditor/equationeditor.php
Образцы разметки: enwiki://Help:Formula
Вольфрамальфа: http://www.wolframalpha.com/
LaTeX pastebin: http://mathbin.net
Математический справочник: http://dxdy.ru/matematicheskij-spravochnik-f52.html
предыдущий тред >>74216
>> No.77311 Reply
>>77310
Лол, и сразу разметку запорол.
>> No.77314 Reply
>> No.77316 Reply
>>77314
В предыдущем треде долго перемывали эту тему, но, кто бы список не сделал, обязательно нашлись бы те, кто посчитал бы его херней. Поэтому решили не париться и вынести в заголовок список вообще не с чанов.
>> No.77418 Reply
>>77409
> Если что - "бесконечно много" не обязательно значит "все".
Здорово подмечено, коллега.
>> No.77421 Reply
>>77316
Добавьте всё-таки Алуффи.
Почитал вот, охуительно же, вроде.
>> No.77436 Reply
Присоединюсь к вам, а то в аутиста превращаться стал, хотя в НМУ даже ходить начал.
>>77314
Тифаретник жив, просто у них DNS-сервер помер, и доменное имя не резолвится. Лечится внесением записи в hosts-файл.
>>77421
Особых профитов всё же нет. Учебник алгебры, как учебник алгебры.
>> No.77479 Reply
>>77421
Добавим. И Арнольда добавим, ибо он крут.
>> No.77491 Reply
>>77479
Не кормите этого.
>> No.77493 Reply
>>77403 (Анону, пытающемуся неспеша разобраться с теорией множеств)
Множества кое-кто называет семействами, классами, и т.п. О теории семейств никогда не слышал. Множество, будь оно произвольным или нет, в рамках ЗФ подчиняется аксиомам ЗФ.
Аксиома объединения:
для любого множества y существует множество x, такое, что для любого z z принадлежит x тогда и только тогда, когда существует w, принадлежащее y, такое, что z принадлежит w.
То есть, зная, что у - множество, мы можем, по аксиоме, утверждать, что нечто (x), содержащее все элементы элементов множества у, тоже является множеством.
с: мыслей курить
>> No.77497 Reply
>>77436
> в аутиста превращаться стал
> в НМУ даже ходить начал
Ты на правильном пути,
хороша твоя дорога.
>> No.77503 Reply
>>77310
Выручай вузач.
Докажи данную теорему как можно подробнее:
Любые две системы действительных чисел изоморфны. При чем изоморфизм определен однозначно.
С меня лучи добра
>> No.77515 Reply
>>77503
Десятичной записи числа одной системы поставь в соотвествие десятичную запись числа другой системы.
А если рассматривать R как множество классов эквивалентности последовательностей Коши из Q, то совсем всё очевидно. Хотя это то же самое.
>> No.77518 Reply
>>77310
отсканьте кто-нибудь листочки по алгебре и анализу пожалуйста
>> No.77519 Reply
>>77518
Могу загрузить так или дать свой FTP для всяких нужд сделал - книжки всякие еще там есть.
>>77515
Речь здесь, наверное, о том, что каждые две модели структуры вещественных чисел, заданные аксиомами, изоморфны. Моделями, например, являются перечисленные бесконечные десятичные дроби и классы эквивалентности фундаментальных последовательностей рациональных чисел.
>>77503
Подробно не буду. Раз изоморфизм (а там структура упорядоченного полного поля), то 1 переходит в 1', а 0 - в 0'. Так можно построить множество целых неотрицательных чисел (0 и 1, 1+1, 1+1+1 - в силу аксиом все существуют и различны). У натуральных есть противоположные, и они тоже переводятся изоморфизмом. У этих есть обратные и они тоже поставлены в соответствие. Так получится, что множество рациональных чисел перешло в множество рациональных чисел (да еще и с подобающим порядком). Далее через полноту доказываем, что на иррациональные изоморфны.
%%А доказывать это надо над конкретной аксиоматикой.
Если честно, то никогда в деталях это рассуждение не проделывал, но когда-нибудь займусь.%%
>> No.77521 Reply
File: 1347918650574.png
Png, 0.93 KB, 300×20 - Click the image to expand
edit Find source with google Find source with iqdb
1347918650574.png
>>77503
В Феферман С., Числовые системы есть. Можешь почитать, но там своя система, поэтому придтся повникать в термины и логические связи между утверждениями. Пересказывать не могу, потому что не охота этим сейчас заниматься.
>> No.77526 Reply
>>77519
> дать свой FTP
давай
>> No.77529 Reply
>>77526
ftp://gjz.no-ip.org/ листок в files1, кодировка UTF-8
>> No.77531 Reply
File: ana_maria_ilinca_04.jpg
Jpg, 52.66 KB, 567×850 - Click the image to expand
edit Find source with google Find source with iqdb
ana_maria_ilinca_04.jpg
>>77519
>>77521
>>77515
Спасибо, няши. Добра вам :3
>> No.77582 Reply
>>77529
всех благ тебе, мил человек
>> No.77585 Reply
>>77493
> О теории семейств никогда не слышал.
Семейство множеств - это множество множеств, все элементы которого являются подмножествами одного какого-то множества.
Класс - это множество множеств.
>> No.77586 Reply
>>77585
> все элементы которого являются подмножествами одного какого-то множества.
Необязательно.
> Класс - это множество множеств.
Любое множество - тоже класс. В теории множеств с классами множество - это класс, являющийся элементом другого класса. В общем, классы не нужны, как и теории множеств с классами.

Так или иначе, всякие "семейства" по причине исторически сложившихся терминов сохранились. Это просто множества.
>> No.77587 Reply
>>77586
Бардак со множествами начался после Гильберта и его дружков, пытавшихся построить всю математику на основе теории множеств. Гильберту сотоварищи обломал рога Гёдель, а бардак остался.
>> No.77588 Reply
>>77587
Во-первых, не вижу никакого бардака. Во-вторых, можно много обсуждать историю и философию, но вся математика действительно построена на теории множеств и равнонепротиворечивых теориях. Неформальных доказательств в математике быть не может - есть только те, формализация которых является простым, но нудным упражнением.
>> No.77910 Reply
>>77587
ЗОЗО, ты штоле?!
>> No.77913 Reply
>>77910
Нет.
>> No.78012 Reply
Ебать, как я ненавижу лекции Вьюгина.
>> No.78013 Reply
>> No.78031 Reply
Анон, нужна помощь, натолкни хотя бы на путь истинный.
A - конечное множество, ассоциативность отношений уже доказана;
x,y,a принадлежат A
ax=ay -> x=y
xa=ya -> x=y
Доказать: A - группа.
>> No.78033 Reply
>>78013
Да.
>> No.78038 Reply
>>78013
Ну да, видимо.
За фихтенгольца в НМУ полагается антиреспект.
Я надеюсь, что второй семестр будет более осмысленный.
>> No.78039 Reply
>>78031
Основные свойства группы помнишь?
1. Ассоциативность
2. Существование нейтрального элемента
3. Наличие обратного элемента
отсюда и танцуй.
>> No.78040 Reply
>>78039
Я не вижу путей доказать существование нейтрального элемента и уж тем более обратного.
>> No.78048 Reply
>>78031
> ассоциативность отношений
Операции же.

Понадобится принцип дирихле. Дальше идет доказательство существования нейтрального элемента:
Пусть A = {X1,X2, .. , Xn}
Рассмотрим произведения:
X1 * X1 = Xk1
X2 * X2 = Xk2
....
Xn * Xn = Xkn
Если существуют Xki = Xkj i=/=j то
X1 * Xn = Xki
X2 * Xm = Xkj =>
X1 * Xn = X1 * Xm =>(по условию) Xn = Xm => i = j противоречие
Значит все ki различны =>(принцип дирихле)
Существует Xp
X1 * Xp = X1
Обозначим Xp за E
X1 * E = X1
Докажем что E двухсторонний обратный к X1
(X1 * E) * X1 = X1 * X1 =>(по ассоциативности)
X1 * (E * X1) = X1 * X1 =>
E * X1 = X1 = X1 * E
Докажем что Е нейтральный:
Рассмотрим произведение:
Xm * E = Xn
(Xm * E) * X1 = Xn * X1
Xm * (E * X1) = Xn ( E X1) =>
Xm = Xn
>> No.78049 Reply
>>78048
> двухсторонний обратный
двухсторонний нейтральный
фаст-фикс
>> No.78051 Reply
Чего-то из того, что ты написал, не доказывается нихуя.
Множество целых чисел {1,2,3} с умножением удовлетворяет тому, что ты написал, но группой не является.
Ты, наверное, забыл упомянуть про то, что бинарная операция действует на множестве А (то есть, любым двум элементам из А сопоставляет элемент из А), иначе нихуя не докажешь.
Непонятно просто, что такое ax и ay.
>> No.78053 Reply
>>78051
Ах да, я предполагал что операция *:AxA -> A думаю ы задании это подразумевалось.
>> No.78059 Reply
Буду считать, что
 (1) А - конечное непустое множество;
 (2) * - функция, D(*)=A*A, R(*)⊆A;
 (3) для любых a, x, y из A (a*x)*y=a*(x*y);
 (4) для любых a, x, y из A a*х=а*у влечёт х=у,
 (5) для любых a, x, y из A х*а=у*а влечёт х=у.
 Тогда можно доказать, что для любого а из А {u: u∈A*A и u=(х, у) влечёт у=а*х} - биекция из А в подмножество А.
 Поскольку множество А конечно, то R({u: u∈A*A и u=(х, у) влечёт у=а*х})=A.
 (6) Тогда для любых а, у уравнение а*х=y имеет корень х.
 (7) Похожим образом можно доказать, что для любых а, у из А в А относительно х разрешимо уравнение х*а=у.
 (8) По (6) для некоторого а имеется такое е из А, что а*е=а.
 (9) Значит, в А существует такое е, что для любого у из А по (7), (3) и (8) у*е=(х*а)*е=х*(а*е)=х*а=у.
 (10) Из (6) следует, что для любого х из А в А существует такое у, что х*у=е.
 Из (1)-(3), (9) и (10) вытекает, что (А, *, е) - группа.
>> No.78060 Reply
>>78053
тогда всё элементарно, вроде.
Если ax = ay -> x=y (умножение слева определено однозначно) то
(aa...a)x = (aa...a)y -> x=y (умножение на а слева n раз),
Так как множество конечно, существует такое n, что:
(аа...а) = а (умножение n раз)
(aa...a)x = ах (умножение n раз),
(аа...a)x = x (умножение n-1 раз) (вот левая единица).
То, что левая единица является также и правой, следует из ассоциативности и однозначности операции, легко доказывается.
Так как а мы можем взять любое, и для каждого найдётся такое n, что:
(аа...а) = 1 (умножение n раз),
то (аа...а)(а) = 1 (умножение n-1 раз в первой скобке) (вот тебе обратный элемент)
>> No.78062 Reply
>>78060
> Докажем что E двухсторонний обратный к X1
> То, что левая единица является также и правой, следует из ассоциативности и однозначности операции, легко доказывается.
Хочу обратит внимание коллег на вынужденную двухсторонность односторонней единицы.
>>78059
>> No.78071 Reply
>>78053
Не подразумевалось. Все так как есть.
>> No.78072 Reply
>>78059
Спасибо, я, кажется, понял суть твоего решения, красиво.
>> No.78079 Reply
>>78071
Тогда это может быть вообще внешняя операция, ставящая в соответствие паре элементов из А китайскую залупу, и мы так ничего не докажем.
>> No.78085 Reply
>>78079
А, прошу прощения, это именно так. Операция внутренняя.
>> No.78108 Reply
>>78059
> Тогда можно доказать, что для любого а из А {u: u∈AA и u=(х, у) влечёт у=ах} - биекция из А в подмножество А.
> Поскольку множество А конечно, то R({u: u∈AA и u=(х, у) влечёт у=ах})=A.
Не понял этого момента, можно подробнее?
>> No.78112 Reply
"Я читал лекцию первокурсникам о проблеме Пуанкаре, которую решил наш известный математик Перельман. Я хорошо знаю эту математику, и там главный ответ на вопрос – как устроена Вселенная. Если набросить веревку на всю Вселенную и тянуть петлю, вопрос: эта веревка затянется без узлов – в точку стянется или появятся узлы на этой веревке? Отсюда следует ответ: если веревка, наброшенная на всю Вселенную, тянется без узлов, значит, Вселенная гомеоморфна сфере, то есть она в каком-то смысле подобна сфере. Если будут узлы, то Вселенная устроена по-другому. Вот проблема Пуанкаре. Перельман доказал, что эта веревка – я говорю на таком языке – не затянется, то есть Вселенная гомеоморфна сфере.

Мы смотрим на звезду, свет падает и мы видим эту звезду. Но если Вселенная в каком-то смысле подобна сфере, то этот свет пойдет и в другую сторону от звезды, он обогнет Вселенную и должен попасть к нам в затылок, значит, мы должны видеть эту звезду и сзади. Астрономы не знают ответа на этот вопрос – я его задавал."

Внимание, вопрос: угадайте с трёх попыток, кто автор этой пасты
подсказка: МГУ находится во второй сотне в рейтинге университетов
>> No.78114 Reply
>>78112
Я не знаю, кто автор, но хуита отменная.
>> No.78115 Reply
File: 110942_0.png
Png, 0.77 KB, 111×50 - Click the image to expand
edit Find source with google Find source with iqdb
110942_0.png
Исследовать на сходимость.
Воспользовался радикальным признаком Коши и... дальше не знаю. В знаменателе степень n-1 / n и что с ней делать я не знаю.
>> No.78116 Reply
>>78114
> но хуита отменная
Склонен с вами не согласиться. В пасте корявым языком описаны здравые идеи. В районе нашей звёздной системы (Солнечной) Вселенная, предположительно, представляет односвязное компактное трехмерное многообразие. Если экстраполировать на всю Вселенную, то по теореме Пуанкаре как раз и получится нечто, гомеоморфное трёхмерной сфере.
> Но если Вселенная в каком-то смысле подобна сфере, то этот свет пойдет и в другую сторону от звезды, он обогнет Вселенную и должен попасть к нам в затылок, значит, мы должны видеть эту звезду и сзади.
Над этим вопросом ещё Эйнштейн бился и даже сотворил знаменитый кащенизм с покраской вселенной.
> Астрономы не знают ответа на этот вопрос – я его задавал.
Вероятно, имелся в виду вопрос о форме вселенной и о том, не являются ли некоторые из наблюдаемых звёзд одним и тем же объектом.
>> No.78121 Reply
>>78116
Хорошо, не отменная, но хуита. Автор почему-то связал какие-то физико-философские проблемы с конкретной математической, хотя Пуанкаре Вселенную вовсе ввиду не имел, когда свою проблему формулировал.
>> No.78122 Reply
>>78121
> Хорошо, не отменная, но хуита
Нет, именно отменная.
Дело в том, что эта паста с точки зрения содержательности никак не изменится, если слово "Вселенная" заменить любым другим, например, словом "жопа".
Просто если бездарю необходимо поддерживать имидж учёного, он должен регулярно произносить на публике какие-то наукообразные слова типа "форма Вселенной" и т.д.
>> No.78123 Reply
>>78112
Таки слышал точно такие же слова в передаче на телеканале культура. Передача была буржуйская.
>> No.78124 Reply
>>78123
Анон, я вот тут делаю алгебру НМУшную, и у меня вопрос, что за поле такое С((z))?
>> No.78125 Reply
>>78124
> С((z))
Формальные ряды Лорана с комплексными коэффициентами, я полагаю
>> No.78126 Reply
>>78125
Спасибо.
>> No.78138 Reply
>>78108
 Для любого M существует такое S, что для любого х тогда и только тогда х∈S, когда существует такое X, что X∈M и x∈Х. Это множество S называется объединением множеств класса M. По аксиоме экстенсиональности объединение единственно. Объединение по классу M буду обозначать ∪X[X∈M].
 Отношением считаю только любое множество W, для любого элемента x которого имеются такие a, b, что x=(a, b).
 D(W) - множество, равное {x: x∈∪X[X∈∪Y[Y∈W]] и  существует такое y, что (х, y)∈W}, R(W) - множество, равное {x: x∈∪X[X∈∪Y[Y∈W]] и  существует такое y, что (y, x)∈W}.
 Функцией считаю только любое такое отношение F, что для любых а, b, c из (a, b)∈W и (a, c)∈W следует, что b=c.
 Любую функцию F тогда и только тогда считаю биекцией из X в Y, когда
 - для любых a, b, c из (a, b)∈F и (c, b)∈F следует a=c;
 - D(F)=X;
 - R(F)=Y.
 Любые A, B тогда и только тогда называю равномощными, когда существует биекция из A в B.
 Любое A тогда и только тогда называю конечным множеством, когда не существует таких его собственного подмножества B и F, что F - биекция из А в В.
 Для любого а из А {u: u∈A*A и u=(х, у) влечёт у=а*х} - отношение, поскольку {u: u∈A*A и u=(х, у) влечёт у=а*х} по смыслу записи является множеством исключительно некоторых пар.
 D({u: u∈A*A и u=(х, у) влечёт у=а*х})=A, так как по смыслу записи для любого х из А в {u: u∈A*A и u=(х, у) влечёт у=а*х} имеется такая пара, для которой х является первым членом.
 R({u: u∈A*A и u=(х, у) влечёт у=а*х})⊆A по смыслу записи {u: u∈A*A и u=(х, у) влечёт у=а*х}.
 Для любых p, s, t из (p, s)∈{u: u∈A*A и u=(х, у) влечёт у=а*х} и (p, t)∈{u: u∈A*A и u=(х, у) влечёт у=а*х} следует, что s=t, поскольку s=a*p=t. Значит, {u: u∈A*A и u=(х, у) влечёт у=а*х} - функция.
 Для любых p, s, t из (p, s)∈{u: u∈A*A и u=(х, у) влечёт у=а*х} и (t, s)∈{u: u∈A*A и u=(х, у) влечёт у=а*х} следует p=t в силу a*p=s=a*t.
 Значит, {u: u∈A*A и u=(х, у) влечёт у=а*х} - биекция из А в R({u: u∈A*A и u=(х, у) влечёт у=а*х}).
 Поскольку множество А конечно, то его подмножество R({u: u∈A*A и u=(х, у) влечёт у=а*х}) не является собственным подмножеством A. Тогда R({u: u∈A*A и u=(х, у) влечёт у=а*х})=A.
>> No.78140 Reply
>>78115
Признака сравнения достаточно: ... < (n^n)/((1,5n)^(n-1)) = 1,5 * n/(1,5^n) и всё сходится.
>> No.78151 Reply
>>78124
Формальные ряды лорана над комплексными числами.
>> No.78170 Reply
>>78122
> Дело в том, что эта паста с точки зрения содержательности никак не изменится, если слово "Вселенная" заменить любым другим, например, словом "жопа"
Зато осмысленность исчезнет.
> если бездарю необходимо поддерживать имидж учёного
Отличай необходимость от достаточности.
>> No.78181 Reply
>>78122
> Нет, именно отменная.
Ты не понял человека, который говорит тебе умную мысль. Вот тебе паста.

Ниже мы попытаемся объяснить теоретическую возможность конечности Вселенной. Пока что заметим лишь, что конечность Вселенной не означает наличие у неё края, “стены”. Ведь само по себе отсутствие у геометрической фигуры конца и края ещё не означает её бесконечности. Поверхность нашей планеты, например, конечна, но края у неё нет. В детстве я, как и другие, наслаждался старинной картинкой, на которой был изображён монах, дошедший до Края Земли и просунувший голову сквозь небесный свод. Ещё более, чем упомянутая картинка, детское воображение увлекала модная гипотеза (потом она как-то заглохла), что некие две далёкие туманности, наблюдаемые с Земли в противоположных концах небосвода, являются на самом деле не различными астрономическими объектами, а одним и тем же объектом, видимым с разных сторон. Если бы это подтвердилось, это было бы доказательством конечности Вселенной. Вот три мысленных эксперимента, способные засвидетельствовать указанную конечность, если она действительно имеет место. Первый: экспериментатор отправляется в космическое путешествие и, двигаясь всё время в одну сторону, возвращается в исходную точку. Второй: обнаруживается окружность, длина которой меньше той, которую сообщают нам в школе, то есть меньше двух пи, помноженных на длину радиуса. Третий (предложен Эйнштейном): экспериментатор окружает себя сферой, сделанной из прочной и неограниченно растягивающейся плёнки, и начинает эту сферу раздувать; площадь поверхности сферы сперва будет возрастать, но начиная с некоторого момента — уменьшаться, а в итоге вся сфера стянется в точку — при том, что экспериментатор остаётся внутри сферы.
Чтобы понять, как такое возможно, надо напрячь воображение, а затем рассуждать по аналогии.
Двумерная сфера - поверхность трёхмерного шара. Вообразим себе обычную двумерную сферу, населённую двумерными же существами; их принято называть флатландцами. Мы с вами живём на сфере (на поверхности Земли), флатландцы же пребывают в теле сферы, в её “толще”; эта “толща”, конечно, не имеет толщины, но ведь и флатландцы её не имеют. Органы чувств не позволяют флатландцам ощутить что-нибудь вне пределов этой сферы, которая для них составляет Вселенную. Сфера большая, а двумерные жители обитают на небольшом её участке и — внимание! — полагают, что их Вселенная представляет собою двумерное евклидово пространство, то есть плоскость. Посмотрим, что может поколебать их в этом убеждении. Если считать, что флатландцы умеют видеть чрезвычайно далеко, то удалённый от них объект они видят с двух сторон: ведь в их Вселенной луч света идёт по сфере, огибая её. Космический путешественник, двигающийся всё время в одну сторону, возвращается, обогнув сферу, в исходную точку. Радиус окружности двумерные существа проводят по сфере, и его длина оказывается больше радиуса той же окружности, проведённого в недоступном им “внешнем” пространстве, — а потому длина окружности окажется меньшей, нежели та, которая вычисляется через “фатландский радиус” по нашей школьной формуле. Посмотрим теперь, что произойдёт, если двумерный экспериментатор окружит себя канцелярской резинкой, способной неограниченно растягиваться, придаст ей форму окружности и станет увеличивать радиус этой окружности. Сперва длина окружности будет возрастать, а после прохождения через “экватор” уменьшаться и в итоге уменьшится до нуля.
А теперь картину, только что изложенную нами для двумерного мира, надо по аналогии перенести на мир трёхмерный. Мы, как и флатландцы, убеждены, что пребываем в “прямом” евклидовом пространстве школьной геометрии. Однако не исключено, что на самом деле — в (не “на”, а “в”) сфере, только трёхмерной. И эту трёхмерную сферу можно представлять себе расположенной в евклидовом четырёхмерном пространстве — наподобие того, как двумерная сфера расположена в пространстве трёхмерном. Четырёхмерного пространства мы, разумеется, не воспринимаем своими органами чувств, но ведь и флатландцы не воспринимают пространства трёхмерного. Как и флатландцы, мы можем убедиться в кривизне мира, увидев какой-нибудь весьма отдалённый предмет с двух противоположных сторон или сравнивая длину окружности с той, которая выражает эту длину через радиус по стандартной, известной из школы формуле.
>> No.78204 Reply
Вот ещё от того же автора:

"Два наших аспиранта мехмата, специалисты по криптографии, несколько месяцев назад установили мировой рекорд. Суть в том, что надо разложить большое число на сомножители двух простых. Они считали несколько месяцев и разложили число 2 в 67-й степени и тем самым установили новый мировой рекорд в криптографии."
>> No.78206 Reply
>>78204
а что за автор всёй-таки?
>> No.78207 Reply
>>78204
Хватит, не делай так больше.
>> No.78213 Reply
>>78204
А, ну я так и знал, что это садовник.
Хотя угадать трудно, потому как в рашинской академической среде таких куча, а то ещё и похуже.
>> No.78215 Reply
>>78204
Лол. А разве это не байка, он действительно такое сказал?
>> No.78218 Reply
>> No.78219 Reply
>>78215
Это из его пресс-конференции или другого публичного выступления, вроде.
>> No.78222 Reply
>>78204
Вот чем на мехмате занимаются.
ТруЪ-математика.
Нужно больше терафлопсов.
>> No.78224 Reply
File: 313549648.jpg
Jpg, 101.48 KB, 1024×729 - Click the image to expand
edit Find source with google Find source with iqdb
313549648.jpg
>>78213
>>78215
>>78206
Свет далёких звёзд, обогнувши вселенную, уже бьёт ему в затылок.
>> No.78227 Reply
>>78204
> Они считали несколько месяцев и разложили число 2 в 67-й степени
Эм. Хм. Гыы... Лол.
>> No.78228 Reply
>>78204
МГУ-кун, а что на самом деле сделали эти аспиранты?
>> No.78234 Reply
>>78224
Ок, ты выиграл. Перестаю его защищать и признаю, что он впал в чинопочитание напополам с маразмом и перестал понимать, чем занимаются научные сотрудники его универа.
Но идею вселенной как трёхмерной сферы таки продолжаю считать интересной.
>> No.78239 Reply
File: macro-читай-фихтенгольца.png
Png, 922.82 KB, 1024×729 - Click the image to expand
edit Find source with google Find source with iqdb
macro-читай-фихтенгольца.png
>> No.78243 Reply
>>78234
>>78234
> перестал понимать
так и не понимал никогда, он же по партийной и общественной линии продвигался

В математичсекой тусовке есть забавная легенда. Она гласит, что когда Садовничему подошёл срок защищать докторскую, на каждую его попытку приходил Арнольд и указывал на ошибки. В резултате пришлось защищаться то ли в другом вузе, то ли вообще в другом городе, не помню уже. Вполне правдоподобно, кстати.
>> No.78246 Reply
>>78243
Ты меня прямо-таки огорчаешь, я был лучшего мнения о мехмате и даже собирался туда в магистратуру поступить.
А в каких отношениях Садовничий с Зоричем?
>> No.78253 Reply
>>78246
> туда в магистратуру
А по какой специальности, если не секрет?
Я вот специально заглянул сюда http://master.math.msu.ru/magisterskie-programmy/
и сильно удивился, на матемктическом факультете нет ни одной математической программы в магистратуре.
>> No.78255 Reply
На кафедре математики обсуждаются задачи физики.
>> No.78257 Reply
>>78243
Точней, говорят, что докторскую за него писала еврейская аспирантка, а садовник не мог ни на один вопрос осмысленно ответить (может, он и понимал её, но с осмысленной речью у него действительно проблемы, как видно выше итт).
Байки байками, а в антисемитизме 60-х - 80-х годов он принимал участие, что чести ему тоже не делает.
Дитя "чёрного 20-летия", впрочем, могло быть и хуже.
>> No.78258 Reply
>>78253
Мехмат - факультет прикладников и программистов потому что, давно уже.
Оно ничего плохого, конечно, но надо тогда просто по-честному не называться "ведущим математическим вузом", а распилиться между ВМиК и физфаком.
>> No.78260 Reply
>>78253
Секрет, ибо деанон локального масштаба.
Что там с Зоричем, почему Садовничий его не выжил, как Арнольда и прочих?
>> No.78264 Reply
>>78260
> почему Садовничий его не выжил
Не знаю совершенно.
Но в каких-либо подлостях Зорич замечен не был и считается заслуженным и приличным человеком.
>> No.78265 Reply
Математиконяши, по слухам один японец доказал какую-то abc-гипотезу. Это правда? Расскажите о ней пожалуйста.
>> No.78266 Reply
>>78260
Выживали только тех, кто много выёбывался.
Кто-то же должен был оставаться, чтобы комсоргам писать докторские.
>> No.78272 Reply
Садовничий Виктор Антонович

Ректор МГУ,член президиума Всемирного Русского Народного Собора (национал-патриотическая организация под эгидой Русской Православной Церкви)
>> No.78276 Reply
>>78272
> Иногда спрашивают: "Ну что, машинка стоит, суперкомпьютер, что она значит?" На мой взгляд как специалиста по информатике, сейчас ни одну сложную задачу в науке и в технологии нельзя решить без суперкомпьютера.
> ...
> Каждая страна старается иметь свои государственные секреты. Для этого надо уметь шифровать, иметь хороших криптографов. А для этого нужно иметь алгоритм шифровки. Та страна надежнее держит свои секреты, у которой более мощная система шифрования. Сейчас в мире идет гонка – кто же рекордсмен? Американцы поставили некоторые задачи несколько месяцев назад – так называемый конфликт хэш-функций. Я говорил об этом даже президенту. По телевидению мелькнуло слово "хэш" - все спрашивают, что это такое? Это оказывается, что разные сообщения ввиду того, что все, конечно, на компьютере могут давать одинаковый набор чисел на выходе. Значит, это дискредитированная шифровка: прочтение разных сообщений попадает в итоге в одинаковую строку. Это конфликт. Есть такая гонка – конфликт хэш-функций. Два наших аспиранта мехмата, специалисты по криптографии, несколько месяцев назад установили мировой рекорд. Суть в том, что надо разложить большое число на сомножители двух простых.
> Они считали несколько месяцев и разложили число 2 в 67-й степени (это что-то такое бесконечное) и тем самым установили новый мировой рекорд в криптографии. Это произвело колоссальное впечатление на все центры - аспиранты мехмата на нашем суперкомпьютере установили такой мировой рекорд. Можно и дальше задачи перечислять. Суперкомпьютеры – это научное могущество государства.
В. А. Садовничий, Академик РАН, Председатель Совета Ректоров России, Председатель Российского Совета олимпиад школьников, ректор МГУ, доктор физико-математических наук, профессор кафедры функционального анализа, профессор кафедры математического анализа.
Робко продолжаю надеяться, что руководитель он намного более грамотный, чем математик.
>> No.78279 Reply
>>78276
Родился на улице Герцена, в гастрономе № 22. Известный экономист, по призванию своему — библиотекарь. В народе — колхозник. В магазине — продавец. В экономике, так сказать, необходим. Это, так сказать, система… эээ… в составе 120 единиц. Фотографируете Мурманский полуостров и получаете te-le-fun-ken. И бухгалтер работает по другой линии — по линии библиотекаря. Потому что не воздух будет, академик будет! Ну вот можно сфотографировать Мурманский полуостров. Можно стать воздушным асом. Можно стать воздушной планетой. И будешь уверен, что эту планету примут по учебнику. Значит, на пользу физики пойдет одна планета. Величина, оторванная в область дипломатии, дает свои колебания на всю дипломатию. А Илья Муромец дает колебания только на семью на свою. Спичка в библиотеке работает. В кинохронику ходят и зажигают в кинохронике большой лист. В библиотеке маленький лист разжигают. Огонь… эээ… будет вырабатываться гораздо легче, чем учебник крепкий. А крепкий учебник будет весомее, чем гастроном на улице Герцена. А на улице Герцена будет расщепленный учебник. Тогда учебник будет проходить через улицу Герцена, через гастроном № 22, и замещаться там по формуле экономического единства. Вот в магазине 22 она может расщепиться, экономика! На экономистов, на диспетчеров, на продавцов, на культуру торговли… Так что, в эту сторону двинется вся экономика. Библиотека двинется в сторону 120 единиц, которые будут… эээ… предмет укладывать на предмет. 120 единиц — предмет физика. Электрическая лампочка горит от 120 кирпичей, потому что структура, так сказать, похожа у нее на кирпич. Илья Муромец работает на стадионе «Динамо». Илья Муромец работает у себя дома. Вот конкретная дипломатия! «Открытая дипломатия» — то же самое. Ну, берем телевизор, вставляем в Мурманский полуостров, накручиваем там… эээ… все время черный хлеб… Дак что же, будет Муромец, что ли, вырастать? Илья Муромец, что ли, будет вырастать из этого?
>> No.78281 Reply
>>78279
Да, вполне возможно, что он слетел с катушек от умственного перенапряжения.
>> No.78337 Reply
File: 14986-115722-2153d8298246ea13344698892148431e.jpg
Jpg, 54.99 KB, 344×445 - Click the image to expand
edit Find source with google Find source with iqdb
14986-115722-2153d8298246ea13344698892148431e.jpg
>>78265
Бамп.
>> No.78344 Reply
>>78337
для начала глянь видца
http://video.yandex.ru/users/fizmatlib/view/1#
>> No.78358 Reply
>>78344
Спасибо, глянул.
>> No.78389 Reply
Вечер, матемач. Помоги пожалуйста разобраться.
Пытаюсь доказать, что ограниченное множество в эрэн с евклидовой метрикой компактно. Делаю так:
Пусть множество ограничено, под этим понимаем, что есть такой шар, в который влезает все множество.
Тогда предположим, что наше множество не компактно, а стало быть, какую последовательность его точек ни возьми - из нее не получится набрать фундаментальную. Значит, расстояние между любыми двумя точками этой последовательности никак не меньше некоторого ε. Возьмем и запилимс открытый шар вокруг каждой точки последовательности радиусом ε/2.
Интуитивно - вот оно, противоречие - внутри шара конечного радиуса уместилось бесконечно много непересекающихся шаров радиуса ε/2.
Теперь вопрос: а туда ли я вообще зашел в рассуждениях? И если да - то как не привлекая меру строго показать противоречие?
>> No.78393 Reply
File: 1310630768495.jpg
Jpg, 31.71 KB, 500×500 - Click the image to expand
edit Find source with google Find source with iqdb
1310630768495.jpg
>>78389
> ограниченное множество в эрэн с евклидовой метрикой компактно
А замкнутость кто будет требовать? А?
> а стало быть, какую последовательность его точек ни возьми - из нее не получится набрать фундаментальную
ёбать-колотить
> какую последовательность его точек ни возьми
Гуглани-ка определение компактности и секвенциальной компактности (там речь о сходящихся подпоследовательностях, а не фундаментальных, хотя в R^n они совпадают), да потом построй к нему отрицание.
> не получится набрать фундаментальную. Значит, расстояние между любыми двумя точками этой последовательности никак не меньше некоторого ε
Неправильно построенне отрицание к определнию фундаментальной последовательности.

Короче, ты везде наврал, одна ошибка охуенне другой просто мне кажется, это кто-то пошутил просто
>> No.78394 Reply
>>78389
> ограниченное множество в эрэн с евклидовой метрикой компактно
Контрпример - (0;1).
>> No.78395 Reply
>>78393
Это вопрос выбора определений же. У меня так - допустим я определяю компакт через сходящиеся последовательности, а множество называю компактным (т.е это разные вещи) если поменять в определнии сходящиеся на фундаментальные.
> Неправильно построенне отрицание к определнию фундаментальной последовательности.
Тут не отрицание к фундаментальности, а отрицание к "из любой можно выбрать фундаментальную подпоследовательность". Здесь, кстати, я порядком сомневаюсь, так что если у тебя есть правильно сформулированное отрицание - я с удовольствием тебя послушаю.
Друзья, я ценю ваше желание помочь, но я вынужден работать вот с этими конкретно определениями. Спасибо.
>> No.78396 Reply
>>78395
> отрицание к "из любой можно выбрать фундаментальную подпоследовательность"
чтобы это ни значило Отрицание к этому "существует такая последоватедовательность из которой нельзя выбрать фундаментальную" Может ты имел ввиду "из любой ограниченной последовательности можно выделить сходящуюся"
>> No.78397 Reply
>>78396
> существует такая последоватедовательность из которой нельзя выбрать фундаментальную подпоследовательность
фикс
>> No.78398 Reply
File: русская_логика.jpg
Jpg, 251.39 KB, 453×700 - Click the image to expand
edit Find source with google Find source with iqdb
русская_логика.jpg
>>78395
> я определяю компакт
> множество называю компактным
> т.е это разные вещи
Ты решил определения переизобретать?
На всякий стлучай: "компакт" на сленге означает "компактное топологическое (под-)пространство", не больше и не меньше.
>> No.78399 Reply
>>78395
> допустим я определяю компакт через сходящиеся последовательности
> поменять в определнии сходящиеся на фундаментальные
Ы. Это ж одно и то же.
>> No.78401 Reply
>>7839
Бывают компактные подмножества и бывают компактные подпространства (терминология!). В действительности одно и то же (легко доказать эквивалентность). Отличие в определениях: у компактного подмножества объединение покрытия содержит его, а у компактного пространства равно ему. Компактные подмножества используются в курсе анализа в некоторых заведениях.
>>78395
Сформулируйте определения и требования, пожалуйста, и прочитайте их. У вас каша в голове.
>> No.78403 Reply
File: field.PNG
Png, 2216.07 KB, 813×930 - Click the image to expand
edit Find source with google Find source with iqdb
field.PNG
Какие факты используются в подчеркнутой части доказательства? Интуитивно это кажется понятным, но хочется разобраться.
>> No.78404 Reply
>>78396
> чтобы это ни значило
Ок, "Из любой последовательности точек, принадлежащих множеству, можно выбрать фундаментальную подпоследовательность."
> Отрицание к этому "существует такая последоватедовательность из которой нельзя выбрать фундаментальную"
Да, я имел в виду именно такое отрицание, проницательный анон.
Далее: возьмем эту самую последовательность. Раз из нее не взять фундаментальной, значит для любой ее подпоследовательности, за исключением конечного числа точек, расстояние между двумя точками не может быть меньше некоторого числа. Все ли верно?
>> No.78405 Reply
>>78403
Неразложимый многочлен порождает простой идеал, в кольце многочленов с коэфициентами из поля все идеалы главные, значит любой простой является и максимальным, т.е. фактор по (f) будет полем. Факторпространтсво имеет размерность d над изначальным полем из p элементов, значит в нём p^d элементов.
Power of Algebra
>> No.78406 Reply
>>78404
Построим отрицание к определению фундаментальной "Существует Е>0, такой что для любого N, существуют x1,x2 что p(x1,x2) > E
Важно помнить что фундоментальность - свойство конца последовательности.
>> No.78407 Reply
>>78406
> существуют x1,x2 > N
фикс
>>78405
спасибо
>> No.78408 Reply
>>78406
> Построим отрицание к определению фундаментальной "Существует Е>0, такой что для любого N, существуют x1,x2 что p(x1,x2) > E
Ах да, я действительно ошибся, спасибо!
> свойство конца последовательности.
Вот тут непонимат.
>> No.78409 Reply
>>78408
> Вот тут непонимат.
Не обращай внимания. Это я решил скопитанить, что у последовательности в начале может быть что угодно и при этом она может быть фундаментальна.
>> No.78415 Reply
>>78246
Зачем тебе магистратура мехмата, когда есть магистратура матфака?
>> No.78435 Reply
>>78406
Вернусь к этой теме.
> Существует Е>0, такой что для любого N, существуют x1,x2 что p(x1,x2) > E
Значит, для любой подпоследовательности нашей последовательности, есть такое число ε, что какой большой номер члена мы бы не взяли - за ним все равно будут две точки, удаленные друг от друга на расстояние, большее ε.
То есть, в любой подпоследовательности будет бесконечно много точек, удаленных друг от друга на расстояние, большее некоторого числа ε. И таким образом, если построить вокруг каждой из этих точек открытый шар радиуса ε/2 - мы получим бесконечно много непересекающихся шаров ненулевого радиуса.
Тут ошибки есть?
>> No.78442 Reply
Аноны, как там в независимом? Толпа подрассосалась?
>> No.78481 Reply
>>78435
Я получаю информацию только из этого поста, выше не читал.
> То есть, в любой подпоследовательности будет бесконечно много точек, удаленных друг от друга на расстояние, большее некоторого числа ε. И таким образом, если построить вокруг каждой из этих точек открытый шар радиуса ε/2 - мы получим бесконечно много непересекающихся шаров ненулевого радиуса.
Коллега, аккуратнее в этой ситуации.
В рассматриваемом подмножестве множества значений последовательности для любой точки есть такая точка, которая удалена от первой более чем на ε, но не гарантируется, что таких точек бесконечно много.
Более того, члены последовательности могут повторяться. Если p(u, v)>ε и для любого натурального числа k имеются такие m, n, которые в смысле натуральных чисел больше k и m имеет образ u и n — образ v, то последовательность может не принимать значений, отличных от u и v.
>> No.78541 Reply
>>78435
> То есть, в любой подпоследовательности будет бесконечно много точек, удаленных друг от друга на расстояние, большее некоторого числа ε.
Да, достаточно взять N1 = max(x1,x2) + 1
> То есть, в любой подпоследовательности будет бесконечно много точек, удаленных друг от друга на расстояние, большее некоторого числа ε
Нет, контрпример: 1,0,1,0,1,0.....
>> No.78542 Reply
>>78541
> Да, достаточно взять N1 = max(x1,x2) + 1
Блин, максимум из номеров x1, x2
>> No.78545 Reply
>>78541
> Нет, контрпример: 1,0,1,0,1,0.....
Из нее можно выбрать фундаментальную 1,1,1.... (по условию этого сделать нельзя)
Но вот тут >>78481 анон очень хорошее замечание сделал - есть только бесконечное множество таких пар x,y что p(x,y) < E, но это ничего не гарантирует, когда речь заходит о точках из разных пар.
Короче говоря, сложновато получается, что-то я не в ту степь полез с этим доказательством.
>> No.78546 Reply
File: DUUDE.jpg
Jpg, 27.04 KB, 500×397 - Click the image to expand
edit Find source with google Find source with iqdb
DUUDE.jpg
>>77310
Математики, разъясните многочлен Лагранжа. Есть формула f(x)=(y1*(x-x2))/(x1-x2) +(y2(x-x1))/(x2-x1), есть точка х=1,135 и две ближайшие х=1,1 и х=1,2. Как найти приближённое значение функции Лапласа Ф(х) в данной точке и определить погрешность?
>> No.78569 Reply
>>77310
Кафедра, ответь на вопрос, пожалуйста.
Принадлежит ли элемент b множеству {a, {b, c}} ?
>> No.78579 Reply
>>78569
Какой хитрый вопрос.
b∈{a, {b, c}} эквивалентно b=a или b={b, c}.
Кстати, когда задаёте вопрос о теории множеств, то указывайте, что за теория множеств. Они-то различаются, в отличии от теорий натуральных чисел или теорий непрерывных полей.
>> No.78586 Reply
>>78569
нет
>> No.78588 Reply
>>78579
> Кстати, когда задаёте вопрос о теории множеств, то указывайте, что за теория множеств
Так я же не знаю, какие бывают. Нуб же, мало что понимаю, разбираюсь, преподавателя спросить не получается. Спасибо. У меня есть ещё два вопроса.
1)Множества {a} и {a,∅} - это одно и то же множество?
2)Вот пусть будет A = {a,b,c}. По свойствам множеств A ⊆ A. То есть {a,b,c} и {a,b,c, A} - это одно и то же множество?
>> No.78592 Reply
>>78588
На оба вопроса в рамках простой (неформальной) принятой мной системы ответ отрицательный.
Пожалуй, я чуть попозже выложу аксиомы.
>> No.78593 Reply
>>78588
1)Нет, пустое множество может быть элементом другого множества.
Т.е. {a} U {∅} = {a,∅} но {a} U ∅ = {a}
2)Разные множества, ты перепутал принадлежность элемента ко множеству со включением одного множества в другое.
>> No.78594 Reply
>>78593
Ок, спасибо.
> Т.е. {a} U {∅} = {a,∅}
А разве не {a,{∅}} ?
>> No.78595 Reply
>>78594
Нет,
∅ - пустое множество, множество без элементов
{∅} - множество, элементом которого является пустое
{{∅}} - множество элементом которого является {∅}
{a} U ∅ = {a}
{a} U {∅} = {a,∅}
{a} U {{∅}} = {a,{∅}}
>> No.78597 Reply
File: 0_110b05_1c92bbd0_L.jpg
Jpg, 26.69 KB, 500×438 - Click the image to expand
edit Find source with google Find source with iqdb
0_110b05_1c92bbd0_L.jpg
>>78595
Ясно.
Спасибо, кафедра, вы няши.
>> No.78600 Reply
File: Феферман-С.-Числовые-системы.zip
Zip, 4.31 KB, 0 files - Click the image to get file
view
Феферман-С.-Числовые-системы.zip
Когда вы пытались угадать, что хотел сказать задававший вопрос анон, то отвечали неверно. Может даже быть, что он нарочно хотел обхитрить кафедру.

>>78597
Аксиомы в Добавлении I в книге. Есть и глава "Теоретико-множественные предпосылки", где даётся интуитивное обоснование теории множеств, а также вводятся некоторые дальнейшие понятия.
(В третьем и четвёртом абзацах Добавления I автор говорит, что всякий рассматриваемый в теории множеств объект - множество или отношение тождества или отношение принадлежности.)
Возможно, это излишний материал, если ты не специализируешься в чистой математике, но он поможет разобраться с простыми понятиями и избежать ошибок и неточностей. Он достаточно сжатый и с классической точки зрения излагает простые сведения из классической системы теории множеств.
Если же ты заинтересован в чистой математике, то это сгодится для небольшого кача на начальном левеле. С другой стороны, может быть, ты предпочтёшь сразу взяться за более абстрактное.
>> No.78604 Reply
>> No.78635 Reply
File: 1255459264662.gif
Gif, 26.66 KB, 282×393
Your censorship settings forbid this file.
unrated
1) доказать, что множество всех действительных чисел из [0,1], в десятичной записи которых цифра 5 встречается бесконечное число раз, является борелевским.
2) доказать, что сумма несчетного числа положительных величин равна бесконечности. моя попытка: обозначим минимальную величину за m, тогда исходная сумма ≥ Σm > счетной суммы Σm = lim(nm)=∞
>> No.78637 Reply
>>78635
алсо, интересно для каких положительных последовательностей 2 тоже справедливо.
>> No.78638 Reply
>>78635
> Σm > счетной суммы Σm
Пруф?
>> No.78639 Reply
>>78635
> сумма несчетного числа положительных величин
Определение скажите, пожалуйста.
>> No.78641 Reply
>>78635

1) Вроде бы просто: рассмотрим дополнение - все действительные числа из [0,1], в десятичной записи которых цифра 5 встречается конечное число раз. Понятно, что это счётное множество.

2) По-моему условие некорректно. Как строго определить "сумму несчетного числа положительных величин"? Ну и в твоём решении m может быть равно 0.
>> No.78642 Reply
>>78641
> Понятно, что это счётное множество.
Почему?
>> No.78644 Reply
>>78635

По поводу 2). Легко доказать, что можно выбрать счётное подмножество, сумма которого равна бесконечности. Будет ли это решением твоей задачи?
>> No.78652 Reply
>>78642

Да я глупость написал. Хорошо, а что если так:
1) Зафиксируем какое-нибудь расроложение конечного числа пятёрок в десятичной записи: 0.x5xx5...
2) Выбросим из [0, 1] все числа, в которых пятёрки есть ещё на каких-то местах (аналогично построению канторова множества) (счётное число шагов)
3) Проделаем 1), 2) для всех возможных расстановок конечного числа 5-к (счётное число вариантов)
4) Объединим то, что получилось. По идее должно получиться дополнение к искомому множеству.
>> No.78673 Reply
>>78638
счетная сумма является подсуммой этой суммы и при этом в ней остается еще несчетное число элементов.
>>78639
с величинами (это же константы?) как-то неинтересно получается.
интереснее с функциями: Σ(по всем точкам x несчетного множества А)f(x), f определена на А и принимает только положительные значения. может ли эта сумма быть конечной?
>>78641
> Ну и в твоём решении m может быть равно 0.
почему? они же все > 0.
>> No.78705 Reply
>>78673
> они же все > 0
Он понимает положительность в смысле Бурбаки.
>> No.78736 Reply
А кто-нибудь расскажет мне, какое определение у "несчётной суммы" и что вообще вы тут несчётно суммируете?
>> No.78740 Reply
>>78736
Возможно, речь о мере идёт.
>> No.78741 Reply
То есть они объединяют множества положительной меры по несчётному классу.
>> No.78745 Reply
>>78736
Сумма несчетного количества элементов.
>> No.78749 Reply
Анон, привет. У тебя случайно нет какой-нибудь вводной статьи о рядах Лорана? Ну или вообще о рядах, но чтобы ряды Лорана разбирались там отдельно.
>> No.78750 Reply
>>78745
эта фраза сама по себе, очевидно, не имеет смысла
>> No.78753 Reply
>>78750
Само по себе ничего смысла не имеет.
Берётся несчётное множество элементов и суммируется,
хуле непонятного?
>> No.78754 Reply
>>78753
Как это записывается формально, несдержанный няш?
>> No.78757 Reply
>>78753
Тебе известно, что ты тупой?
а) В алгебраических структурах определены только конечные суммы, произведения и т.д.
б) Сумма счётного множества называется суммой ряда. О рядах можно говорить только в нормированных пространствах, кольцах и т.д., то есть где определна сходимость оп норме.
в) Для определения "несчётной суммы" существует наука, которая называется теория меры, а несчётная сумма в этой науке называется интегралом.
Вот я и пытаюсь понять, о чём тут идёт речь (а заодно выяснить, понимают ли сами участники беседы о чём они говорят).
>> No.78760 Reply
>>78757
Умник нашёлся, блять.
Норма-хуёрма...
Фихтенгольца читал хоть?
>> No.78762 Reply
Анон, скажи мне, а верно ли, что счётное объединение нигде не плотных в каком-то множестве подмножеств нигде не плотно (не надо доказательства, я сам это вроде доказал уже, просто решал задачу и пришёл к этому).
>> No.78763 Reply
>>78762
Ах, да, блять.
Самое главное забыл.
Полное по метрике множество.
>> No.78767 Reply
>>78760
Зря ты так, ведь дело говорит. Простейший пример, дающийся всем первокурсникам, показывающий что нельзя просто так бесконечно суммировать.
Рассмотрим ряд (-1)^n
С одной стороны:
сумма (-1)^n = 1 + (-1 + 1) + (-1 + 1) + ... = 1
С другой:
сумма (-1)^n = (1 - 1) + (1 - 1) + ... = 0
Обычно поступают так. За Sn обозначают сумму первых n членов ряда, и если Sn->a n->бесконечности говорят что а - сумма ряда. а может быть бесконечностью (!)предела может и не быть вообще.
В контексте выше сказанного сказать что такое сумма несчетного числа слагаемых нельзя.
>> No.78770 Reply
File: 397px-John_Forbes_Nash,_Jr._by_Peter_Badge.jpg
Jpg, 51.41 KB, 397×599 - Click the image to expand
edit Find source with google Find source with iqdb
397px-John_Forbes_Nash,_Jr._by_Peter_Badge.jpg
>>78760
И откуда только вы понабежали?..
>> No.78784 Reply
>>78767
Абсолютно сходящиеся можно. Вообще пример не в тему.
>> No.78790 Reply
>>78784
Наркоман, сумма ряда хоть и абсолютно сходящегося это предел частичных сумм и только он, как определить частичные суммы для несчетного числа слагаемых - хз.
>> No.78816 Reply
>>78635
> сумма несчетного числа
Произнеси все последовательности слов русского языка.
> равна бесконечности
А ты никогда не пытался искать в совокупности кроликов аккумуляторы?
>>78749
Если для НМУ, то там ничего не нужно.
>>78762
Нет, это не правда, но на теорему Бэра похоже. Контрпример: множество рац. чисел.
>>78757
> Сумма счётного множества
Wut?
>> No.78819 Reply
File: ava.png
Png, 34.19 KB, 112×112 - Click the image to expand
edit Find source with google Find source with iqdb
ava.png
>>78757
Ну ты и дурачок.
>> No.78820 Reply
>>78762
Не понял. Множество рациональных чисел - счетное объединение нигде не плотных множеств.
>> No.78824 Reply
Привет, матемамач. Это опять я со своими последжовательностями.
Я задался вопросом - а какими вообще свойствами обладает последовательность, из которой нельзя извлечь фундаментальную подпоследовательность?
Вот возьмем, допустим, саму последовательность, она нефундаментальна. Значит нашлось ε1 такое, что какие бы большие номера в последовательности мы ни брали - за ними все равно будет бесконечное множество пар точек с номерами вида n,n+1, удаленных на расстояние, большее ε1.
Извлечем все эти точки, получим подпоследовательность, которая согласно условию также нефундаментальна.
Значит найдется некоторое ε2, такое что -- аналогичные рассуждения, в конце также извлекаем пары точек --
Бесконечно продолжаем процесс, получим числовую последовательность {εn}. Можно ли что-то сказать о ее сходимости?
>> No.78825 Reply
>>78824
Ну, кроме того, что снизу она ограничена нулем, конечно же.
>> No.78827 Reply
>>78824
> получим числовую последовательность {εn}
По построению, какие бы большие номера в последовательности мы ни брали - за ними все равно будет бесконечное множество пар точек с номерами вида k,k+1, удаленных на расстояние, большее εn.
Она не фундаментальна.
>> No.78831 Reply
>>78827
Ох, нет, няша, я тут имел в виду последовательность из самих чисел ε. Мы берем для n-й подпоследовательности все ее ε, выбираем из них минимальное и назвыаем его εn.

Есть подозрение, что она сходится, так как - εn <= ε(n-1) по построению - монотонна и ограничена.
>> No.78834 Reply
>>78824
Ох, до чего хренью любят здесь страдать.
> пар точек с номерами вида n,n+1, удаленных на расстояние, большее ε1.
Если про расстояние между точками в паре, то, очевидно, врёте, сэр.
Всё довольно просто. Если множество значений последовательности находится во вполне ограниченном множестве, то с необходимостью такая последовательность содержит подпоследовательность, являющуюся фундаментальной. Если взять инъективную последовательность, то для того, чтобы среди её подпоследовательностей не было фундаментальных, необходимо и достаточно, чтобы в шаре любого радиуса с любым центром лежало лишь конечное число образов натуральных чисел.
А вообще сам вопрос бессмыслен: метрические свойства множеств хорошо классифицированы, последовательности рассматривать незачем.
>> No.78836 Reply
И да, аноны, возвращаясь к совсем давнему вопросу - как доказать, что в шар некоторого радиуса R не может поместиться бесконечно много непересекающихся шаров меньшего радиуса?
Я не знаю теории меры, но вроде это утверждение сразу из аксиом следует, нет?
Как-нибудь, не привлекая меру, это утверждение для R^n с еквлидовой метрикой доказать можно?
>> No.78837 Reply
>>78836
> как доказать, что в шар некоторого радиуса R не может поместиться бесконечно много непересекающихся шаров меньшего радиуса?
Может же. В (0;1) например.
>> No.78839 Reply
>>78837
Няш, опиши чуть подробнее, я не понял.
>> No.78841 Reply
>>78839
Если я правильно тебя понял, нужно поместить в шар некоторого известного радиуса бесконечно много шаров с меньшим радиусом. Имеем шар (0;1), разделим его "пополам" на шары (0;0.5) и (0.5;1), шар (0.5;1) снова разделим пополам и так до бесконечности.
>> No.78842 Reply
>>78841
Ох, прости, не все сказал - шары одинаковые.
>> No.78843 Reply
>>78834
> Если множество значений последовательности находится во вполне ограниченном множестве, то с необходимостью такая последовательность содержит подпоследовательность, являющуюся фундаментальной.
С этим фактом я знаком и вполне сознательно его в данном случае не замечаю.
> Если взять инъективную последовательность, то для того, чтобы среди её подпоследовательностей не было фундаментальных, необходимо и достаточно, чтобы в шаре любого радиуса с любым центром лежало лишь конечное число образов натуральных чисел.
Вот за это спасибо.
> незачем
Но любопытно же!
> Если про расстояние между точками в паре, то, очевидно, врёте, сэр.
Где именно вру?
>> No.78848 Reply
>>78843
> С этим фактом я знаком и вполне сознательно его в данном случае не замечаю.
Но он же определяющий!
> Вот за это спасибо.
А тут я наврал. Нужно брать фиксированный радиус.
> Где именно вру?
Контрпример: 1, 1+1/2, 1+1/2+1/3, ..., и для каждого положительного числа найдется номер, что между последующим и предыдущим расстояние менее его, но... Или еще x_n=n^.5.
>> No.78849 Reply
>>78836

В произвольном пространстве - неверно. Например, бесконечные последовательности l_2. На каждой координатной оси можно построить шарик.

В R^n: допустим можно. Построим конечную epsilon-сеть так, чтобы любой шар из бесконечного множества содержал в себе какую-то точку этой сети (выбрать достаточно маленькое epsion). Противоречие: оказывается, шаров можно втиснуть не более, чем точек epsilon-сети.

Аналогично для любого пространства, где шар - компактное множество.
>> No.78850 Reply
File: M.png
Png, 13.25 KB, 446×73 - Click the image to expand
edit Find source with google Find source with iqdb
M.png
Решил попробовать себя на извилистом пути мудрости. Начал с Куранта Робинса «Что такое математика?».
По пикрилэйтед — первое сделал левой пяткой, а второе — сообразить не могу, хотя доказательство, видимо, имеет маленький состав по числу пунктов.
То ли не проснулся ещё толком, то ли совсем не уметь. Хотя Сканави прорешивал, несколько доказательств из Евклидовой геометрии помню.
Реквестирую каких-нибудь простеньких книг по построению математических доказательств.
>> No.78852 Reply
>>78850
Я не понял условие 2). Вот, например, сравнение иного вида:
1^2 + 2^2 + ... + (p-1)^2 = 0 (mod p) при p > 3
>> No.78853 Reply
>>78852
Точнее, требуется нет p > 3, а НОД(p, 6) = 1
>> No.78854 Reply
>>78850
При p - простом, Z/pZ - область целостности, поэтому из [x]^2-[a]^2=[0]=>[x]=[a] или [x]=-[a]=[p-a], что и требовалось. Глупая задачка.
>> No.78856 Reply
>>78848
> Контрпример
Ай-ай-ай, как неловко-то!. Ну вот, а говоришь, что незачем - так я хоть определение фундаментальной последовательности знать буду. Да, действительно, был неправ, про номера их нечего сказать. Но по крайней мере хоть в том, что их бесконечно много вроде не наврал.
> Но он же определяющий!
Ну да, факт замечательный. Но вот только я им в силу поставленного вопроса пользоваться не могу. Я пытаюсь (к сожалению, предыстория по треду разбросана) из органиченности в R^n доказать предкомпактность (секвенциональную), ну или саму вполне ограниченность.
>>78849
Спасибо, бро. Теперь мне осталось доказать, что шар в R^n вполне ограничен. Буду думать.
>> No.78857 Reply
>>78816
Лол, я туда даже и не хожу.
>> No.78860 Reply
File: лифт.jpg
Jpg, 8.84 KB, 280×280 - Click the image to expand
edit Find source with google Find source with iqdb
лифт.jpg
Помогите разобраться.
Задача:
Лифт отправляется с 6 пассажирами и останавливается на 10 этажах. Найти вероятность того, что никакие два пассажира не выйдут на одном этаже.
Мое решение:
1 способ (если не различать пассажиров):
есть 6 пассажиров и 9 разделителей => всего способов выйти С6(6+9)=С615
способов выбрать каждому по разному этажу = С6_10
итого вероятность = С610/С615=(10!9!)/(15!4!)
2 способ (если различать пассажиров):
каждый пассажир может выйти на одном из 10 этажей, поэтому всего способов 10^6
способов выбрать каждому по разному этажу = А6_10
итого вероятность = А6_10/10^6=10!/(4!10^6)

Видим, что вероятности разные, хотя учет различия пассажиров не должен ведь повлиять на ответ.
Где ошибка?
>> No.78864 Reply
File: Безымянный.png
Png, 68.16 KB, 1864×975 - Click the image to expand
edit Find source with google Find source with iqdb
Безымянный.png
Анон, ответь срочно пожалуйста, правильно решил? Интеграл расходится?
>> No.78872 Reply
>>78864
Вроде правильно.
>> No.78875 Reply
>>78860
> учет различия пассажиров не должен ведь повлиять на ответ
В данном случае влияет. Разбери тот же самый пример, когда 2 этажа и 2 пассажира.
>> No.78896 Reply
>>78872
А меня вот отчислили, мол неверно.
>> No.78898 Reply
File: 500px-Sinx_x_limit_proof.svg.png
Png, 18.02 KB, 500×500 - Click the image to expand
edit Find source with google Find source with iqdb
500px-Sinx_x_limit_proof.svg.png
>>78896
Нет, точно расходится.
Если не доверяешь асимптотике, то можно легко доказать, что
tg(x) >= x (при 0 <= x < pi/2), следовательно tg(1/x^2) >= 1/x^2 при x >= 1

Например, на пикрелейтед длина дуги AK = x, то AL = tg(x)
Площадь сектора круга OAK = x/2, площадь треугольника OAL = tg(x)/2
Очевидно, что SOAK <= SOAL, значит x <= tg(x)
>> No.78899 Reply
>>78898
Возможно, ответ правильный, а решение неправильное?
>> No.78900 Reply
>>78899
Может ты условие неправильно переписал?
Решение правильное, tg(1/x^2) при x->infty мало отличается от 1/x^2
А может препод хотел проверить твою уверенность в решении? У нас был один такой. Иногда рандомно говорил "неверно", хотел, чтобы его убедили. Ну или просто у него было задание тебя отчислить :3
>> No.78903 Reply
>>78899
>>78900
Все условие как на пике, метод решений тот же. Только внятнее и аккуратнее.

У меня депрессия, добрач.
>> No.78905 Reply
>>78903
Не расстраивайся. Конечно, зря довёл до такого состояния. В академ уйти нельзя?
Можешь поработать пока, восстановиться или перепоступить (надеюсь, у тебя есть отмаза от армии)

Мамке скажи, что на доброчане подтвердили твою правоту, лол
>> No.78907 Reply
File: 1334867287282.jpg
Jpg, 1012.63 KB, 2560×1600 - Click the image to expand
edit Find source with google Find source with iqdb
1334867287282.jpg
>>78905
Отмаза есть, в академ нельзя. Егэ пересдавать снова не охота, все подзабыл, хотя там для даунов. Даже не знаю что теперь делать.
>> No.78942 Reply
>>78896
Где учишься?
Может, оно и к лучшему, что отчислили?
>> No.78981 Reply
File: Снимок.PNG
Png, 7.32 KB, 710×55 - Click the image to expand
edit Find source with google Find source with iqdb
Снимок.PNG
Как решать, анон? Теория чисел, лол
>> No.78984 Reply
>>78981
Что такое (a_1, ... , a_n)?
>> No.78986 Reply
>>78984
НОД этих чисел.
>> No.78988 Reply
>>78981
Я не разбираюсь в теории чисел, но рискну предположить, что по индукции. Для одного натурального числа такое лямбда существует, оно равно единице. Значит, для полного доказательства утверждения нужно как-то показать, что если лямбды существуют для n натуральных, то и для n+1 будут существовать, но как это сделать, я не разумею.
>> No.78992 Reply
>>78981
Идеалы, порождённые числами a1, a2 ... an, содержатся в идеале, порождённом их общим НОД.
раздели все числа на НОД, среди них, очевидно, найдутся взаимно простые (пусть они, например, а1 и а2).
По китайской теореме об остатках, двумя взаимно простыми числами можно линейно над Z выразить единицу (и, очевидно, любое целое вообще). Выражай единицу любыми двумя взаимно простыми, умножай равенство на НОД.
>> No.78993 Reply
>>78992
Ой, блять.
Не китайская теорема об остатках, а соотношение Безу (хотя китайская теорема и есть его обобщение).
>> No.78994 Reply
Привет, анон, я пришел с первого курса НМУ, и у меня категорически не получается следующая задача. Необходимо доказать, что если непрерывное преобразование комплексной плоскости с бесконечно удаленной точкой переводит обобщенные окружности в обобщенные окружности, то это преобразование является либо дробно-линейным, либо комплексно сопряженным к дробно-линейному. Может, подскажешь что-нибудь?
>> No.79005 Reply
Няши, чему равен предел
(1+1/2)(1+1/4)(1+1/8)...(1+2^n) n -> беск. ? Пытался оценить, да вот только не получается никак.
С вольфрамом тоже никак, т.к. не могу вывести аналитическое выражение.
>> No.79008 Reply
File: gcd.pdf
Pdf, 0.07 KB, 595×842 - Click the image to get file
gcd.pdf
>>78992
> раздели все числа на НОД, среди них, очевидно, найдутся взаимно простые (пусть они, например, а1 и а2)
не обязательно: 6, 10, 15

Мне нравится такое доказательство.
>> No.79009 Reply
>>79005
Qpochhammer[-0.5,0.5,infinity]
>> No.79010 Reply
>>79005
Равен 2.
Домножь эту хуйберду на (1-1/2) и зацени, как там все схлопнется.
>> No.79011 Reply
>>79010
Нет, схлопнулось бы, если (1+1/2)(1+1/4)(1+1/16)...(1+2^2^n)
>> No.79013 Reply
>>79005
Это ряд SUM([n/2]/2^n), n=0,1,2...
Если тебе это чем-то поможет.
>> No.79014 Reply
>>79013
Скобочки - целая часть.
>> No.79015 Reply
>>79013
Поправка: SUM([(n+1)/2]/2^n), n=0,1,2...
>> No.79016 Reply
>>79010
Спасибо тебе. Большое большое.
>> No.79017 Reply
>>79013
И тебе спасибо.
>> No.79018 Reply
>>79016
Всё-таки вопрос был про ряд (1+1/2)(1+1/4)(1+1/16)...(1+2^2^n)?
>> No.79019 Reply
>>79015
То, что ты написал, сходится к 4/3:
http://www.wolframalpha.com/input/?i=sum%28floor%28%28n%2B1%29%2F2%29+%2F+2^n%29
>> No.79020 Reply
File: x_4fa6b898.jpg
Jpg, 38.38 KB, 500×500 - Click the image to expand
edit Find source with google Find source with iqdb
x_4fa6b898.jpg
>>79005
n=2 в твоем случае.
НЕ БЛАГОДАРИ
>> No.79021 Reply
>>79020
Не получилось запостить.
Короче, здесь http://mathworld.wolfram.com/InfiniteProduct.html - формула (53)
>> No.79022 Reply
>>79015
Еще поправка: 1+SUM([(n+1)/2]/2^n), n=0,1,2...
>> No.79023 Reply
Что-то не то у вас всех.
Эта штука равна чему-то вроде 2.3842...
Мимо взял и посчитал ручками
>> No.79025 Reply
>> No.79045 Reply
>>78204
Он небось хотел сказать 2 в 67-й степени минус 1?
>> No.79046 Reply
>>79045
Нет, это тоже бред. Просто он не в теме, т.к. администратор, а не учёный.
И хоть старый уже, но вцепился в свой пост, как клещ.

Вот про "разложение на множители": http://forumlocal.ru/showthreaded.php?Cat=&Board=University&Number=110[...]&vc=1

Вот ещё недавние новости, если кто не слышал: http://spb.rbc.ru/topnews/28/09/2012/671853.shtml
У меня от этого разрыв второго рода, ЕВПОЧЯ
>> No.79059 Reply
>>79046
> т.к. администратор, а не учёный.
Он учёный. Академик, замдекана мехмата и автор серии учебников по математике.
>> No.79078 Reply
>>79045
Даже если бы он сказал 2^67-1 было бы странно, т.к. разложить это на множетели тоже довольно просто.
>> No.79079 Reply
>>79078
Во-первых, не разложение на множители, а распараллеливание алгоритма де Канниери и Рехбергера подбора хеша.
Во-вторых, не месяц, а пять часов.
В-третьих, не весь суперкомпьютер, а только кусочек.
>> No.79080 Reply
Анон, мне кажется, что я всё-таки тупой.
Я не могу построить математическое доказательство, но хочу научиться.
Чем бы упороться, чтобы хорошо строить доказательства даже из, кхм, книжек «начального» уровня?
Могу решать любой пример школьного уровня или из ЕГЭ, даже С6, но не могу настоящую математику. Может, это дело опыта? За какое время ты справляешься со средним доказательством? Есть ли смысл в нахождении доказательства, но за несколько часов сидения и втыкания в экран?
Получающий-среднее-техническое-кун
>> No.79081 Reply
анон, помоги. некоторое время занимаюсь программированием.
захотелось мне понять как правильно программировать, как грамотно строить алгоритмы, как не путаться в больших в программах и как такие писать. скачал несколько книг, гуглил статьи и получил баттхерт. авторы пользовались непонятным значками, словами и прочей хуйней. позже я узнал, что это был матан! из всего вашего матана отлично знаю арифметику, больше не помню нихую. начать хочется с алгебры, но не знаю куда идти и чего читать, хочется интересную книгу/видеокурс по элементарной алгебре. сперва думал обычные учебники, но меня пугает следующий стиль обучения:"это отнимите, здесь местами поменяйте и для усвоения материала 100500 шаблонных задач.” слышал, что математика должна мышления развивать, нахуй мне такие алгоритмы? Вообщем, анон, что интересного и полезного читать/смотреть по алгебре для того нуба?
>> No.79082 Reply
>>79080
> За какое время ты справляешься со средним доказательством?
Чтобы научиться доказывать теоремы, нужно сидеть и доказывать теоремы, блджад. Если ты в своей жизни не доказал пару тысяч теорем, то над каждой новой будешь сидеть по много часов и тупить. Изучай готовые доказательства. Например, для лоу-левельых алгебраических - скури книгу Э. Ландау "Основы анализа". Для настоящей математики - скури "Теорема Абеля в задачах и решениях". Прорешай г-на Демидовича, у него есть много качественных задач.

Мутация в математика - дело это далеко не быстрое. За месяц упорного труда по два часа в день сформируешь математическое зрение, за год перестроишь мозг под математическое мышление.
>> No.79084 Reply
>>79081
> Вообщем, анон, что интересного и полезного читать/смотреть по алгебре для того нуба?
Ван дер Варден. "Алгебра".
> из всего вашего матана отлично знаю арифметику
Мордкович, учебник алгебры за 10-11 классы.
Зорич, учебник анализа в двух томах.

Ссылки в предыдущем треде.
>> No.79085 Reply
>>79084
ахуеть, прям сразу с анализа?
>> No.79086 Reply
>>79085
Сначала со школьного учебника.
>> No.79087 Reply
>>79082
Спасибо.
>> No.79092 Reply
>>79059
Не хочу спорить. На мой взгляд, то, что его давно пора сменить - это факт.

"Мы стали выдавать собственные дипломы - очень большие, красивые дипломы Московского университета. И уже как следствие - конкурс в Московский университет повысился очень существенно в этом году. Таким образом, наша учебная часть, академическая часть программы развития, на мой взгляд, реализуется очень хорошо"

"У нас работает шестьсот коллективов и двести стоит в очереди, - похвалился Садовничий. - Причем на этом суперкомпьютере получаются в том числе и рекордные результаты. Например, создано новое лекарство, которое поступило в аптеки. Без суперкомпьютера его нельзя было создать, это некие глазные капли"

Кстати, по слухам эти капли он пиарит, т.к. сам имеет долю. http://skq-project.ru/doc/index.php?ID=52

Ну и фееричное
"Недавно я создал комиссию, в которую входили около 30 академиков - ученые, которые работают в зарубежных университетах, или вернулись, или часто бывают в этих университетах. Это математики номер один в мире. Я попросил их сравнить уровень преподавания математики в любом ведущем университете мира - Гарварде, Сорбонне и нашем МГУ. Причем я просил таблицу, перечень курсов, которые читались, и часы. Потом мы провели семинар с докладами этих экспертов, и общий вывод следующий - если сложить все курсы всех ведущих университетов мира и выстроить в один ряд и курсы, читаемые на мехмате, то та сумма составляет две трети от мехмата. Типичная ситуация - половина - если один вуз. Это если говорить о сумме новых курсов.

Нашелся всего один курс из сотен, который читается в одном зарубежном университете и не читается в МГУ - теория Галуа - известного математика, это теория очень специальная. Она читается у нас коротко. Сейчас мы проведем такое сравнение по физике, биологии, механике. Более того, в ряде американских университетов научные семинары идут на русском языке, поскольку все участники вышли из нашей страны"

Конечно, возможно, что он сам во всё это верит. Но одно дело, если бы он был каким-нибудь блаженным старичком-профессором, а он всё-таки ректор и вице-президент РАН.
>> No.79096 Reply
>>79092
> Например, создано новое лекарство, которое поступило в аптеки
Ага, глазные капли. Их пиарят уже много лет, торгуют ими в трёх аптеках (аптека МГУ, какая-то интернет-аптека и ещё одна), этими каплями создатель капель "вылечил" себе катаракту.
> то, что его давно пора сменить
На кого?
>> No.79100 Reply
File: 2432_v-samom-dele.jpg
Jpg, 33.70 KB, 569×492 - Click the image to expand
edit Find source with google Find source with iqdb
2432_v-samom-dele.jpg
>>77310
О чём эта картинка?
>> No.79101 Reply
>>79100
Операция коммутирования, определенная по некоторой ассоциативной операции, удовлетворяет тождеству Якоби.
>> No.79109 Reply
>>79100
Проверяют тождество Якоби для дифференцирований некоего кольца А, что и написано в формулировке предложения.
Это из квадроохуенной книжки "Гладкие многообразия и наблюдаемые".
>> No.79115 Reply
>>79100
"вот почему бурбакистская мафия, заменяющая понимание науки формальными манипуляциями с непонятными „коммутативными“ объектами, так сильна во Франции, и вот что угрожает и нам в России"
>> No.79150 Reply
>>79115
Арнольд говорил не про алгебру как таковую (он сам участвовал в семинарах Бурбаки), а про непонимание сути математики. Например, изучение в учебном пособии умножения натуральных чисел без примеров в виде подсчёта яблок или вычисления площади прямоугольника - бурбакизм, а доказательство в специальной литературе (подчёркиваю, не в популярных учебниках, а в специальных) коммутативности умножения исходя из аксиом Пеано - не бурбакизм.

Короче, если учишь математику, но понятия не имеешь, как её использовать для бытовых задач или в других науках - только тогда говорим о бурбакизме. То есть бурбакизм - это философская концепция. Бурбакизм - это восприятие математики как игры ума, никак не связанной со всем прочим миром.
>> No.79152 Reply
>>79150
> Бурбакизм - это восприятие математики как игры ума, никак не связанной со всем прочим миром.
Хуже, на самом деле. В данном случае имеется в виду бездумное следование аксиомам и чуть-ли не алгоритмически-символьным, то есть абсолютно бессодержательным манипуляциям с, как они считают, "произвольно выдуманными объектами и аксиомами".
>> No.79153 Reply
File: ZenkinMIFI-2005.rar
Rar, 0.01 KB, 0 files - Click the image to get file
view
ZenkinMIFI-2005.rar
Аноны, в приложенной статье некий г-н Зенкин из РАН доказывает противоречивость теоремы Кантора.
Есть ли изъян в его доказательстве?
>> No.79154 Reply
File: Бурбаки.JPG
Jpg, 99.54 KB, 665×643 - Click the image to expand
edit Find source with google Find source with iqdb
Бурбаки.JPG
>>79152
> бездумное следование аксиомам и чуть-ли не алгоритмически-символьным
Почему же чуть ли? Именно что символьным.
>> No.79162 Reply
>> No.79164 Reply
>>79153
Как же я люблю такие статьи!
http://www.trinitas.ru/rus/doc/0016/001c/1499-kl.pdf
>> No.79185 Reply
>>79164
А причина одна - безблагодатность математичсекое невежество.
Все ебанаты от математики на удивление похожи: переносят математические утвержденния на бытовой контекст, разнообразие только в выборе отверждений. Так рождаются отрицатели отрицательных/иррациональных/комплексных чисел, философствование вокруг теоремы Гёделя, опровержение теоремы Пифагора топологией (это, вообще говоря, уже нетривиальная глупость, человек где-то слышал о топологии), сумматоры несчётного множества чисел и т.д.
>> No.79191 Reply
>>79185
> сумматоры несчётного множества чисел
Что ж с этим не так? Такая сумма расходится.
>> No.79192 Reply
>>79191
с ней не так то, что она не определена.
В математической науке для того, чтобы что-то делать, необходимо сначала определить 1) объект с которым ты что-то делаешь 2) собственно, саму операцию
>> No.79193 Reply
File: ar.pdf
Pdf, 0.14 KB, 612×792 - Click the image to get file
ar.pdf
>>79192
> с ней не так то, что она не определена.
Лекции университета Дюка говорят об обратном.
An uncountable sum of positive real numbers always diverges.
Пдфрилейтед.
>> No.79198 Reply
>>79193
спасибо! наконец-то я увидел хоть какое-то определение.
Только оно, конечно, неправильное. В том месте, где "Clearly, this agrees with the usual notion of sum when I is countably infinite" ошибка.
Например,
1 - (1/2) + (1/3) - (1/4) + ... = ln(2),
а у них такой ряд расходился бы.

Всё, наверное, работает для только положительных чисел, но у них в определнии это не указано (там написано c : I → R, т.е. все действительные числа без ограничения)
Вообще по теории меры есть более осмысленные тексты, чем лекции университета Дюка
>> No.79200 Reply
Но идея исчерпывания конечными суммами очень хорошая, конечно, например, интеграл Лебега так определяется.
>> No.79204 Reply
File: zenkin.jpg
Jpg, 11.19 KB, 212×290 - Click the image to expand
edit Find source with google Find source with iqdb
zenkin.jpg
>>79153
> Попытка прямого определения мощности континуума в рамках канторовского
Вот отсюда начинается чепуха.
Но...
> Анищенко И.Г., Зенкин А.А., Зенкин А.А. Семантическая симметрия духовного пространства храма и универсума монадологии Лейбница: неожиданные пересечения искусства и науки
> Семантическая симметрия духовного пространства храма
Лол.
Он, к счастью, коньки отбросил в 2006.
>> No.79228 Reply
>>79192
А ещё, иметь такие операции и сделать для каждой операции.
>> No.79230 Reply
Анон, мне говорят, что имеются формулы классической формализации пеановской системы натуральных чисел, которые истинны в этой теории, но не выводимы в ней.
А как же теорема Гёделя? Ведь наличие истинной в теории невыводимой формулы противоречит ей.
Анон, пожалуйста, помоги мне разобраться.
>> No.79233 Reply
>>79230
Пусть имеем теорию, достаточно сильную, чтобы сформулировать аксиомы Пеано.
По первой теореме Гёделя о неполноте, в ней существует невыводимая и неопровержимая формула.
По второй, формула, утверждающая непротиворечивость этой теории, невыводима в этой теории.
> которые истинны в этой теории, но не выводимы в ней
Пример, пожалуйста.
> Анон, мне говорят, что имеются формулы классической формализации пеановской системы натуральных чисел
Пример?
>> No.79251 Reply
>>79233
Я не знаю конкретных примеров истинных невыводимых формул. Прочитал в книге "Математическая логика" Шенфилда на 84-й странице:
> Таким образом, мы получаем истинную формулу теории N, которая не является теоремой теории N.
N - формализации пеановской арифметики.
>> No.79255 Reply
>>79152
> В данном случае имеется в виду бездумное следование аксиомам и чуть-ли не алгоритмически-символьным, то есть абсолютно бессодержательным манипуляциям с, как они считают, "произвольно выдуманными объектами и аксиомами"
Но ведь это очевидным образом нелепо. Математика состоит в осмысленном конструировании утверждения B и последующем доказательстве того, что B следует из некоторого утверждения A. Нельзя опускать осмысленность конструирования, иначе мы получим машину для написания книг из Академии Прожектёров.
>> No.79257 Reply
>>79255
Это игра. Если мне интересно, я играю в неё. Что в этом плохого?
>> No.79258 Reply
>>79257
Но ведь математика - не только доказательство утверждений, но ещё и формулирование утверждений.
>> No.79259 Reply
File: Шенфилд-Дж.-Математическая-логика.zip
Zip, 3.19 KB, 0 files - Click the image to get file
view
Шенфилд-Дж.-Математическая-логика.zip
>>79251
Прикладываю файл.
>> No.79267 Reply
Утверждение. Каждое натуральное число имеет какое-то интересное свойство.
Доказательство. Пусть n0 - наименьшее число, не имеющее интересных свойств, но разве это не интересное свойство?
>> No.79273 Reply
>>79267
Вот ты шутишь, а кое-кто из этого нестандартный анализ вывел.
>> No.79279 Reply
Реквестирую книжку по теории групп, желательно на Русском, но можно и на английском.
1)Не очень большую <300 страниц.
2)С упражнениями.
3)Не очень сложную. Основы теории групп я знаю, т.е. знаю много определений но почти не знаю теорем и не самых очевидных следствий из них. Хочется этот пробел заполнить.
>> No.79280 Reply
>>79279
Курош?
>> No.79282 Reply
File: холл-и-ленг.zip
Zip, 17.28 KB, 0 files - Click the image to get file
view
холл-и-ленг.zip
>>79279
Холл, Теория групп; С. Ленг, Алгебра.
>> No.79283 Reply
>>79282
> Холл, Теория групп
Судя по всему - то что нужно.
>>79280
Слишком большая книга, и я не нашел упражнений в ней. Буду ей дублировать Холла.
>> No.79416 Reply
Простите что не тему. Здравствуйте. В этом треде вероятно есть люди куда более умные чем я. Я бы хотел узнать как связана тонкая "способность к точным наукам и логике" с такими характеристиками как память, степень концентрации, рабочая память. То есть например, насколько хорошо без всяких мнемотехник и хитрых приемов(кроме самого малого) вы:
1. Можете проводить арифметические операции в уме.
Насколько длинными числами вы можете оперировать? Какой объем информации можете удерживать в голове?
2. Как быстро вы запоминаете?
Например ряд цифр из 9-12 знаков.
3. Насколько хорошо вы концентрируетесь, сколько времени можете провести в наивысшей концентрации?
4. Насколько вы проницательны в реальной жизни?
>> No.79419 Reply
>>79416
Здесь не умные, здесь математики, лол.
> Насколько длинными числами вы можете оперировать?
Свободно складываю-вычитаю пятизначные, девятизначные складываю со скрипом. Нужно бы потренироваться.
> Как быстро вы запоминаете? Например ряд цифр из 9-12 знаков.
5 секунд, полминуты паузы, 2 секунды на повторение, минута, повторение. Хватает примерно на сутки.
> сколько времени можете провести в наивысшей концентрации?
Когда как. Обычно - две-три минуты, потом начинаю осознавать, что существую, и концентрация ломается. За интересными делами могу сидеть по несколько дней.
> Насколько вы проницательны в реальной жизни?
Не знаю, хикка жешь.
>> No.79426 Reply
>>79416
1. Нет, с трудом. Вообще меня сложно заставить думать.
Двузначными свободно. Не знаю.
2. Медленно запоминаю. Во время теста с врачом выдавал кривую запоминания 5-6-6-7-7, что характерно для легко слабоумных и апатичных больных, насколько мне известно. Мне кажется, у меня был инсульт, поэтому я первому понятию соответствую, но апатия тоже есть.
3. Я не могу концентрироваться уже давно. В обычном состоянии расконцентрирован. Кветиапин повышает концентрацию. Интуиции нет.
4. Совершенно не проницателен. Самому аж смешно.
Я не знаю, как спрошенное относится к математике. Книжки я и сейчас читать с объективным пониманием могу (субъективного нет).
>> No.79430 Reply
>>79416
Лучше спроси как "способность к точным наукам и логике" связана с заболеванием неврозом. Я наблюдаю ярко выраженную положительную корреляцию. Клинической картине самых способных математиков, с которыми я пересекался (т.е. лучшие российские математики младше 30 лет), место в палате мер и весов по статье "невроз".
Потом удивляются, что математики долго не живут.
>> No.79433 Reply
>>79430
> с заболеванием неврозом
А такого нет. Раньше же как было: где полегче - невроз, где посложнее - психоз. Сейчас широкий спектр невротических расстройств.
>> No.79437 Reply
>>79430
Нормальные здоровые люди мало способны к математике, увы. У нормальных здоровых людей нет способности к абстрагированию.
>> No.79438 Reply
>>79437
Заблуждение на заблуждении.
>> No.79441 Reply
File: UncSum.png
Png, 95.07 KB, 709×357
edit Find source with google Find source with iqdb
UncSum.png
File: UncSum2.png
Png, 31.48 KB, 869×433
edit Find source with google Find source with iqdb
UncSum2.png

Да, невнимательность проявил автор, ошибся он.
В любом случае, ты неправ в том, что определения нет – оно есть. Более правильное определение из Fremlin D. (Measure Theory, vol. 2), по sum over arbitrary index sets. На этом же основании мы суммируем по несчётному множеству эл-в. Вот, например, определение.
И ещё одно для неупорядоченных сумм по несчётному множеству эл-в.
>> No.79444 Reply
>> No.79445 Reply
>>79441
Можешь поделиться книжками? :3
>> No.79448 Reply
>>79430
Связываю интерес к математике с ОКР. Справки о ОКР не имею, о наличии у себя сужу по википедии и треду в /rf/.
>>79433
Кажется, ты прав.
>> No.79452 Reply
File: Dudley-Real-Analy...
Pdf, 5.71 KB, 367×619
Dudley-Real-Analysis-and-Probability.pdf
File: applied_analysis.pdf
Pdf, 3.39 KB, 612×792
applied_analysis.pdf

>>79445
Да, держи.
>> No.79454 Reply
>>79452
Спасибо.
>> No.79455 Reply
>>79416
> 1. Можете проводить арифметические операции в уме.
> Насколько длинными числами вы можете оперировать? Какой объем информации можете удерживать в голове
Дорогой друг, для математиков эти навыки совершенно бесполезны. Для этого использую компьютеры. Я на своём мехмате на 3-м курсе на практических занятиях числа больше 3-х вижу далеко не всегда, и 3 на 2 не всегда умножаю правильно, но экзамены мне это на 5 сдавать не мешало. На них я вообще почти ничего не считал. А на иных практических занятиях мы вообще на компе программы пишем вместо того, чтобы считать. Многие вычисления нереально без компьютера просчитать. См. Численные методы.
> 2. Как быстро вы запоминаете?
> Например ряд цифр из 9-12 знаков.
Почти тот же ответ. Обчно, чтобы запомнить число, мне нужно им где-то воспользоваться или где-то его увидеть несколько раз в разное время. Я помню очень мало чисел, и запоминаю их просто ужасно. 12 цифр я буду учить кучу времени. У числа е я даже не помню и одной цифры после запятой! Я помню только, что оно между 2 и 3 и ближе к 3, чем к 2. Но зато я точно помю его определение и точно помню, как вычислять его численно через ряды.
> 3. Насколько хорошо вы концентрируетесь, сколько времени можете провести в наивысшей концентрации?
С утра и до вечера. Концентрируюсь превосходно, если нет сильных тревог, мешающих заниматься. Я действительно профессиональное качество, мне кажется.
> 4. Насколько вы проницательны в реальной жизни?
Откуда я знаю? Ты б ещё спросил, насколько я умный.
>> No.79456 Reply
>>79430
> Потом удивляются, что математики долго не живут.
Щито?
>> No.79457 Reply
>>79455
> У числа е я даже не помню и одной цифры после запятой!
Что, серьёзно?
>> No.79459 Reply
>>79455
> но экзамены мне это на 5 сдавать не мешало
Что как бы говорит нам о качестве этих экзаменов.
>> No.79460 Reply
>>79459
Выгоняют часто с нашего факультета между прочим. А зачем мне эти цифры? И почему я должен именно две цифры запомнить? А может одну? Или три? Ты хоть представляешь себе бессмысленность запомнинания цифр иррационального числа? Их там бесконечно много так между прочим, а вот память у меня, как и время моей ничтожной математической жизни конечно. Но определение, как вычислять, что оно обозначает и представляет из себя, где использовать - это я знаю хорошо.
>> No.79461 Reply
>>79460
В самом деле, зачем тебе эти цифры? Чем ты занимаешься?
>> No.79462 Reply
>>79461
Ну у меня правда ещё специализация - дискретка, алгебра, кодинг, графы, автоматы с граматиками и тому подобное. может если б я был ближе к дифурам и матану, я бы обратил на цифры e больше внимания. Ну посмотрю их где-нибудь, если прям нужны будут.
>> No.79463 Reply
>>79462
Печально.
>> No.79464 Reply
Товарищ >>79455 во всём прав. Сам хотел написать то же самое, да поленился.
> У числа е я даже не помню и одной цифры после запятой!
А я помню, что там, кажется, семёрка! Или не семёрка. Херъ бы с нею.

>>79441
спасибо!
>> No.79465 Reply
>>79463
Ну что печального?
>> No.79466 Reply
>>79464
> А я помню, что там, кажется, семёрка! Или не семёрка.
Оки. Можете без гугла вычислить пять-шесть знаков после запятой?
>> No.79467 Reply
>>79466
ряд-то для функции e^x я помню крепко
(хотя и это не принципиально, конечно)
>> No.79468 Reply
>>79467
Оки, вычисли.
>> No.79469 Reply
>>79466
Это займёт время. Думаю, да. Но заниматься я этим щас не буду.
>> No.79470 Reply
>>79467
>>79469
Напоминаете студентов-кодеров, которые с апломбом заявляют "программистам матан не нужен".
Пардон.
>> No.79471 Reply
>>79416
> 1. Можете проводить арифметические операции в уме.
Более менее быстро сложение 2х значных чисел, с большим трудом умножение 2х значных, сложение 4х значных.
> 2. Как быстро вы запоминаете?
Немогу запомнить практически ничего не связанного с уже известным, например путаю правые/левые модули, вообще все правое/левое.
> 3. Насколько хорошо вы концентрируетесь, сколько времени можете провести в наивысшей концентрации?
Пол часа, но только если удастся сконцентрироваться.
> 4. Насколько вы проницательны в реальной жизни?
Сильно зависит от ситуации.
>>79430
> Лучше спроси как "способность к точным наукам и логике" связана с заболеванием неврозом.
Незнаю есть ли у меня "способности", но в какой-то форме психологические заболевания есть.
>> No.79472 Reply
>>79470
Дело в том, что оно им действительно не нужно, лол. В том виде, как это преподаётся математикам, т.е. с полными доказательствами. Да и физикам не нужно. Это по словам самих кодеров и физиков, есличо.

Я. Б. Зельдович написал замечательный учебник «Математика для начинающих физиков и инженеров». Математики пришли в ярость и устроили битвы из-за его (якобы) нестрогости и ошибочности. В конце концов главный критик Понтрягин написал свой (скучнейший) учебник математического анализа для начинающих. В предисловии он заявил, что некоторые физики считают, что можно грамотно пользоваться анализом, не восходя до исчерпывающего исследования его обоснований. И добавил: «Я с ними согласен».

Зельдович был обижен:

— В таких случаях цитируют оппонента, — говорил он. — А так эта цитата — плагиат.

В учебнике Зельдовича производная определялась как «отношение приращения функции к приращению аргумента, в предположении, что последнее достаточно мало».

Что касается меня, то есть вообще довольно много математических вещей, незнанием которых я, конечно, не горжусь, но радуюсь, что не имею лишнего мусора в голове.
>> No.79473 Reply
>>79469
Я вычислил 5 первых членов ряда, умножая и деля на калькуляторе и мне открылись правильно 3 цифры. По-хорошему там надо дальше считать, пока в результате досчитывания первые 6 цифр не перестанут меняться. Но считать больше 5-ти членов я как-то желания не имею.
>> No.79474 Reply
>>79472
> Дело в том, что оно им действительно не нужно, лол.
Обоснуй, пожалуйста.
>> No.79475 Reply
File: 04102012016.jpg
Jpg, 318.66 KB, 2592×1944 - Click the image to expand
edit Find source with google Find source with iqdb
04102012016.jpg
>>79473
Если поделить на калькуляторе дробь в нижнем правом углу, то можно увидеть число е с 3-мя правильными цифрами.
>> No.79476 Reply
>>79455
> для математиков эти навыки совершенно бесполезны
Нет, я и не говорю что это особо нужно для математики.
Я не психиатр, не биолог, и в этом плане вообще никто, но просто я скромно пытаюсь разобраться своими силами кое-в чем.
Просто мне интересно вот что. Есть некий технарь, способный к математике.
При этом известно, что кора головного мозга ограничена в мощности(есть даже аварийное (в случае перегруза) торможение). И хочу определить в этом случае связь между силой самой коры мозга(оперативная память, способность к концентрации, и т.д) и наличием такой способностей к математики.
И если окажется, что в чем-то кроме математики он окажется достаточно посредственным(хоть и не глупым!) то это подтверждает существование такого явления как "математический склад ума", то есть в этой сфере подсознание лучше помогает обладателю такого склада, делает его более проницательным и догадливым именно в этой области. Похоже к этому все и идет. Если это так, то такой склад врожденный и вызвать его сложно или вовсе невозможно, как бы ты не прокачивал память и внимание. Если что, мне только 17 лет поэтому еще не все очевидные вещи я познал. Но я ведь не дурак, да?
Я правильно понял, как это все устроено?
Вчера кстати умудрился после неудачных попыток кое-как записать 12-значное число, после того как я смотрел на него секунд 5. Особо счастливым себя не почувствовал.
>> No.79478 Reply
>>79472
Я кстати от учебников по физике всегда много бугурта испытываю, так как тамошняя логика мне кажется дырявой. А ещё у меня физфаковец вызвал бугурт, сказав, что линейную алгебру он называет геометрией.
>> No.79479 Reply
>>79475
Формула немного неправильная в правом верхнем углу, да.
>> No.79480 Reply
>>79476
Нас преподы учат на контрольных поменьше считать в уме и всё писать, даже самое простое, так как знают, что мы всё сделаем, а 3 на 2 умножим неправильно. кто-то даже сказал, что мы мыслим не так, чтобы считать в уме хорошо. Я слышал вроде о каких-то людях, которые хорошо считать умеют, умножая числа двухзначные в уме, но имеют проблемы с психикой, сумасшедшие, но как ты видешь, это не о математиках.
>> No.79481 Reply
>>79474
ты цитату не полностью привёл, там ещё "В том виде, как это преподаётся математикам, т.е. с полными доказательствами".
Я сам не физик и не кодер, ретранслирую их практически коллективное мнение, о котором узнал при общении и из текстов.
Компутерсайнтисты, например, что по некоторым разделам математики им рассказывают полные доказательства (особенный баттхёрт у них вызывают неконструктивные доказательства), но уйма затраченных на понимание сил оказывается совершенно бесполезна в их практике.

>>79476
просто вспомнилась фраза "математикой занимаются не те, кто имеет к ней талант, а те, кто имеет желание заниматься"
>> No.79482 Reply
>>79455
> Дорогой друг, для математиков эти навыки совершенно бесполезны.
По этой фразе я сразу понял, что сейчас его понесёт.
> 3 на 2 не всегда умножаю правильно
Наркоман, но темы это не касается.
> Я помню очень мало чисел, и запоминаю их просто ужасно. 12 цифр я буду учить кучу времени. У числа е я даже не помню и одной цифры после запятой! Я помню только, что оно между 2 и 3 и ближе к 3, чем к 2. Но зато я точно помю его определение и точно помню, как вычислять его численно через ряды.
Вот тебя про память и спросили, а в качестве шкалы ввели запоминание чисел. Тебя занесло, дружок, и ты так и не ответил.
>> No.79485 Reply
>>79482
> Вот тебя про память и спросили, а в качестве шкалы ввели запоминание чисел. Тебя занесло, дружок, и ты так и не ответил.
Нет, меня спросили про запоминание чисел. Я запоминаю числа с большим трудом.
>> No.79486 Reply
>>79485
Что касается всего остального, смотря что. Что-то быстро, что-то - нет. Я быстро и надёжно запоминаю, как мне кажется, как-то структурированную сложную информацию, или касающуюся чего-то, что я уже знаю, либо как-то меня волнующую. А имена и числа - с трудом.
>> No.79488 Reply
Я, кажется, являюсь «метисом» между «гуманитарием» и «технарём». Интернет-тесты Они есть чепуха, но всё же. показывают одинаковый уровень развития и «технических», и «гуманитарных» способностей.
Пока я начинающий, но мне нравится математика — примером может служить состояние эйфории, если я решил какую-либо задачу или понял доказательство теоремы. Мои конспекты по различным предметам «сокращаются» математикой. Я часто «оковываю» математикой свои рассуждения — из этого предпочтения исходит дополнительная мотивация изучить математику и, во всеоружии, рассмотреть какую-нибудь психологию, например.
Помню большинство физических констант в виде a.bc*10^y, а «е», например, равным 2.718281828459045. Со сложением и умножением двух- и трёхзначных справляюсь отлично, с запоминанием больших объёмов информации, если предварительно их «уалгоритмизировал» — прекрасно. Трёхмерные фигурки представляю хорошо. Помню степенной ряд двойки где-то до 32 степени — очень мне он нравится. Но концентрация у меня, некоторые говорят, хреновая. Как бы её вообще измерить? Иногда в голову влетают какие-то мысли, о которых я сейчас, казалось бы, и не думал — например сегодня, пока отписывался в одном из тредов, просветлился мыслей совершенно посторонней, не относящейся к теме треда.
>> No.79502 Reply
>>79476
Я секунд за 15 запомнил 12-значное число три дня назад. Помню чётко его до сих пор.
Но это не связано с математикой и подходом к ней, просто память врождённая хорошая, плюс иногда, совсем редко, делал для неё "зарядки".

Более того, большинство математиков, как и в любой другой профессии - узкоспецализированные, обычные люди. А многие ещё и дурачки, плохо понимающие, но что-то там знающие
>> No.79504 Reply
>>79481
Не согласен. Неполным знанием впору ограничиваться лишь инженерам.
>> No.79510 Reply
>>79504
Никто сегодня не знает все разделы, вкючая, все факты, все специализируются на чем-то конкретном. Говорят, последними математиками всех разделов были Гильберт и Пуанкаре
>> No.79516 Reply
>>79510
А возможно ли это вообще сейчас?
>> No.79517 Reply
Охуеть, в тред набижал гуманитарий и всех зотролел.
Люблю очень гуманитариев.
>> No.79518 Reply
>>79510
> Никто сегодня не знает все разделы
"Со здешними математиками говорить ни о чём нельзя, так как каждый из них хочет всех учить и не хочет ничему сам учиться. В результате каждый из них разбирается только в одной узкой области и ничего не понимает вне её. Есть специалист по теории тепла [Фурье], есть — по теории упругости [Пуассон], есть - по небесной механике [Лаплас], и только один Коши мог бы что-нибудь понять, но он интересуется только своим приоритетом" - Абель, во время жизни в Париже.
>> No.79519 Reply
>>79517
> в тред набижал гуманитарий и всех зотролел
Не обольщайся, холивору "математик - это не калькулятор"/"не умеешь считать - не понимаешь математику" уже несколько веков.
>> No.79520 Reply
>>79518
Наверно имелось ввиду что с начала 20го века больше нет математиков сделаших крупные открытия в нескольких разных областях математики. наверняка есть контрпример
>> No.79521 Reply
File: Станислав-Смирнов...
Jpg, 82.68 KB, 800×533
edit Find source with google Find source with iqdb
Станислав-Смирнов.JPG
File: Владимир-Воеводск...
Jpg, 27.82 KB, 306×400
edit Find source with google Find source with iqdb
Владимир-Воеводский.jpg

>>79520
> контрпример
>> No.79522 Reply
>>79519
Вспомнился старинный анекдот. У французского математика(есть причины, почему французского, то есть это о французском математическом образовании какого-то там времени, но я не знаю точно) спросили:"Чему равно 2 + 3?". А он ответил:"2 + 3 = 3 + 2, так как сложение комутативно."
>> No.79523 Reply
>>79520
Не знаю, скорее именно наличие знаний во всех областях. Открытие в данной области может прийти из знания совершенно другой. И несколько разных - это же ещё не все сразу.
>> No.79524 Reply
>>79521
> Владимир Воеводский
> Оставаясь более или менее нормальным, не считая того, что я пытался обсуждать происходящее со мной с людьми, с которыми обсуждать этого наверно не следовало, я за несколько месяцев приобрел очень немалый опыт видений, голосов, периодов, когда отдельные части моего тела мне не подчинялись и множества невероятных случайностей. Наиболее интенсивным был период в середине апреля 2007 когда я провел 9 дней (7 из них в мормонской столице Salt Lake City), ни разу за все эти дни не заснув.
> Почти с самого начала я обнаружил, что многие из этих явлений (голоса, видения, различные сенсорные галлюцинации), я могу контролировать. Поэтому я не был испуган и не чувствовал себя больным, а воспринимал все происходящее как что-то очень интересное, активно пытался взаимодействовать с теми "существами" в аудиторном, визуальном а потом и тактильном пространствах, которые появлялись (сами или по зову) вокруг меня
Как считаете, он реально видит скрытый от других людей мир?
>> No.79526 Reply
>>79522
Тфу! Не математика, оговорился, у школьника. Я сегодня наркоман.
>> No.79527 Reply
>>79524
У психологов вроде теория есть, что человека можно считать психочески здоровым, если что-то что он видит или слышыт ему не ничем мешает. Я думаю, шизофреник какой-то как и Нэш(которому это не мешает вас пережить, ха-ха!). Только раздражает, что это так на слуху.
>> No.79537 Reply
Осталось только поговорить о s- и j- типах мышления.
>> No.79542 Reply
>>79526
> Не математика, оговорился, у школьника.
Вот ещё одна интересная история от того же историка.

Рискуя быть понятым одними только математиками, я приведу всё же примеры ответов лучших кандидатов на профессорскую должность математика в университете в Париже весной 2002 года (на каждое место претендовало 200 человек).

Кандидат преподавал линейную алгебру в разных университетах уже несколько лет, защитил диссертацию и опубликовал с десяток статей в лучших математических журналах Франции.

Отбор включает собеседование, где кандидату предлагаются всегда элементарные, но важные вопросы (уровня вопроса "Назовите столицу Швеции", если бы предметом была география).

Итак, я спросил: "Какова сигнатура квадратичной формы xy?"

Кандидат потребовал положенные ему на раздумье 15 минут, после чего сказал: "В моём компьютере в Тулузе у меня есть рутина (программа), которая за час-другой могла бы узнать, сколько будет плюсов и сколько минусов в нормальной форме. Разность этих двух чисел и будет сигнатурой — но ведь вы даёте только 15 минут, да без компьютера, так что ответить я не могу, эта форма ху уж слишком сложна".

Для неспециалистов поясню, что, если бы речь шла о зоологии, то этот ответ был бы аналогичен такому: "Линней перечислил всех животных, но является ли берёза млекопитающей или нет, без книги ответить не могу".

Следующий кандидат оказался специалистом по "системам эллиптических уравнений в частных производных" (полтора десятка лет после защиты диссертации и более двадцати опубликованных работ).

Этого я спросил: "Чему равен лапласиан от функции 1/r в трёхмерном евклидовом пространстве?"

Ответ (через обычные 15 минут) был для меня поразительным; "Если бы r стояло в числителе, а не в знаменателе, и производна требовалась бы первая, а не вторая, то я бы за полчаса сумел посчитать её, а так — вопрос слишком труден".

Поясню, что вопрос был из теории эллиптических уравнений типа вопроса "Кто автор "Гамлета"?" на экзамене по английской литературе. Пытаясь помочь, я задал ряд наводящих вопросов (аналогичных вопросам об Отелло и об Офелии): "Знаете ли Вы, в чём состоит закон Всемирного тяготения? Закон Кулона? Как они связаны с лапласианом? Какое у уравнения Лапласа фундаментальное решение?"

Но ничего не помогало: ни Макбет, ни Король Лир не были известны кандидату, если бы шла речь о литературе.

Наконец, председатель экзаменационной комиссии объяснил мне, в чём дело: "Ведь кандидат занимался не одним эллиптическим уравнением, а их системами, а ты спрашиваешь его об уравнении Лапласа, которое всего одно — ясно, что он никогда с ним не сталкивался!"

В литературной аналогии это "оправдание" соответствовало бы фразе: "Кандидат изучал английских поэтов, откуда же ему знать Шекспира, ведь он — драматург!"

Третий кандидат (а опрашивались десятки их) занимался "голоморфными дифференциальными формами", и его я спросил: "Какова риманова поверхность тангенса?" (об арктангенсе спрашивать я побоялся).

Ответ: "Римановой метрикой называется квадратичная форма от дифференциалов координат, но какая форма связана с функцией "тангенс", мне совершенно не ясно".

Поясню опять образцом аналогичного ответа, заменив на этот раз математику историей (к которой более склонны митрофаны). Здесь вопрос был бы: "Кто такой Юлий Цезарь?", а ответ: "Цезарями называли властителей Византии, но Юлия я среди них не знаю".

Наконец, появился вероятностник-кандидат, интересно рассказывавший о своей диссертации. Он доказал в ней, что утверждение "справедливы вместе А и B" неверно (сами утверждения А и В формулируются длинно, так что здесь я их не воспроизвожу).

Вопрос: "А всё же, как обстоит дело с утверждением A самим по себе, без В: верно оно или нет?"

Ответ: "Ведь я же сказал, что утверждение "A и В" неверно. Это означает, что A тоже неверно". То есть: "Раз неверно, что "Петя с Мишей заболели холерой", то Петя холерой не заболел".

Здесь моё недоумение опять рассеял председатель комиссии: он объяснил, что кандидат — не вероятностник, как я думал, а статистик (в биографии, называемой CV, стоит не "proba", a "stat").

"У вероятностников, — объяснил мне наш опытный председатель, — логика нормальная, такая же, как у математиков, аристотелевская. У статистиков же она совершенно другая: недаром же говорят "есть ложь, наглая ложь и статистика". Все их рассуждения бездоказательны, все их заключения ошибочны. Но зато они всегда очень нужны и полезны, эти заключения. Этого статистика нам обязательно надо принять!"

Специалиста по голоморфным формам тоже одобрили. Довод был ещё проще: "Курс голоморфных функций нам читал (в элитарной Высшей Нормальной Школе) знаменитый профессор Анри Картан, и там римановых поверхностей не было!" — сказал мне председатель. И добавил: "Если я и выучился римановым поверхностям, то только двадцать лет спустя, когда они мне понадобились для работы (в финансовой математике). Так что незнакомство с ними — отнюдь не недостаток кандидата!"

В Московском Университете такой невежда не смог бы окончить третий курс механико-математического факультета. Римановы поверхности считал вершиной математики ещё основатель Московского Математического общества Н. Бугаев (отец Андрея Белого). Он, правда, считал, что в современной ему математике конца XIX века начали появляться не укладывающиеся в русло этой старой теории объекты — неголоморфные функции действительных переменных, являющиеся, по его мнению, математическим воплощением идеи свободной воли в той же мере, в какой римановы поверхности и голоморфные функции воплощают идею фатализма и предопределённости.

В результате этих размышлений Бугаев послал молодых москвичей в Париж, чтобы они выучились там новой "математике свободной воли" (у Бореля и Лебега). Эту программу блестяще выполнил Н. Н. Лузин, создавший по возвращении в Москву блестящую школу, включающую всех основных московских математиков многих десятилетий: Колмогорова и Петровского, Александрова и Понтрягина, Меньшова и Келдыш, Новикова и Лаврентьева, Гельфанда и Люстерника.

Между прочим, Колмогоров рекомендовал мне впоследствии выбранную себе Лузиным в Латинском квартале Парижа гостиницу "Паризиана" (на улице Турнефор, недалеко от Пантеона). Во время Первого Европейского Математического Конгресса в Париже (1992) остановился в этой недорогой гостинице (с удобствами на уровне XIX века, без телефона и так далее). И престарелая хозяйка этой гостиницы, узнав, что я приехал из Москвы, сейчас же спросила меня: "А как там поживает мой старый постоялец, Лузин? Жалко, что он давно не навещал нас".

Через пару лет гостиницу закрыли на ремонт (хозяйка, вероятно, умерла) и стали перестраивать на американский лад, так что теперь этот островок XIX века в Париже уже не увидишь.

Возвращаясь к выбору профессоров 2002 года, замечу, что все перечисленные выше невежды получили (у всех, кроме меня) самые хорошие оценки. Напротив того, был почти единодушно отвергнут единственный, на мой взгляд, достойный кандидат. Он открыл (при помощи "базисов Грёбнера" и компьютерной алгебры) несколько десятков новых вполне интегрируемых систем гамильтоновых уравнений математической физики (получив заодно, но не включив в список новых, и знаменитые уравнения Кортевега-де Фриза, Сайн-Гордон и тому подобное).

В качестве своего проекта на будущее кандидат предложил также новый компьютерный метод моделирования лечения диабета. На мой вопрос об оценке его метода врачами он ответил совершенно разумно: "Метод сейчас проходит апробацию в таких-то центрах и больницах, и через полгода они дадут свои заключения, сравнив результаты с другими методами и с контрольными группами больных, а пока эта экспертиза не проведена, и есть только лишь предварительные оценки, правда, Хорошие".

Отвергли его с таким объяснением: "На каждой странице его диссертации упомянуты либо группы Ли, либо алгебры Ли, а у нас этого никто не понимает, так что он к нашему коллективу совершенно не подойдёт". Правда, так можно было бы отвергнуть и меня, и всех моих учеников, но некоторые коллеги думают, что причина отклонения была иной: в отличие от всех предыдущих кандидатов, этот не был французом (он был учеником известного американского профессора из Миннесоты).
>> No.79552 Reply
>>79510
Я лишь о неполном знании (без доказательств и восхождения к основам) конкретной теории или раздела, который изучаешь, продвигаешь и пользуешься. Это неприемлимо для хорошего физика-теоретика (в отличие от инженера, у них другие задачи). А достаточно хорошие физики ещё и попутно развивают и способствуют развитию мат. формализма - Фейнман, Дирак, Виттен, к примеру.
>> No.79556 Reply
>>79521
контрпример как раз неудачный.
Смирнов занимается исключительно эргодическими вещами, а Воеводский доселе занимался разными теориями когомологий и смежными вещами, т.е. условно говоря, гомологической алгеброй. Сейчас у него серьёзный проект по основаниям математики, но он пока в зародышевой стадии.
Из ныне практикующих математиков универсалом можно назвать Юрия Ивановича Манина, например (есть куча других, конечно, но Манин первый вспомнился).
>> No.79559 Reply
>>79552
А, ну да.
>> No.79609 Reply
>>78994
Эх ты, анон, а я на тебя надеялся. Сам уже доказал.
>> No.79613 Reply
>>79609
Как?
>> No.79614 Reply
>>79609
Извини, что не помог. Мне стыдно, анон.
>> No.79619 Reply
Кстати, тут вообще еще остались люди, которые в НМУ ходят?
>> No.79621 Reply
>>79619
Ну есть, а что?
>> No.79624 Reply
Кстати, раз уж тут такая дискуссия была про запоминание и умножение цифор, то ответь, анон, еще и на такой вопрос - а должно ли уметь ловко преобразовывать выражения? Ну там, всякие суммы сворачивать, замены угадывать, на единицу домножать, ну ты понел.
>> No.79626 Reply
>>79624
Это уже вопрос намного сложнее и интереснее. Человек, который их применяет обычно какбы знает много преобразований по названиям и знает при каких условиях их можно применить и к какому хорошему результату это приведёт и думает, где что можно сделать и к чему таким образом выйти. Это совершенно точно очень полезно в любой математике и скорее всего без этого ничего не придумаешь нового. А вот применение на практике готовых математических теорий уже может этого не особо требовать(если ты инженер, кодер-разработчик ПО, учёный из другой науки, который формулирует задачу на математическом языке и отдаёт её на откуп маетематикам), но тоже очень полезно. Ещё вопрос, насколько это врождённо или этому можно научиться. Я почти уверен, что этому нужно учиться, как минимум, а может и одного обучения недостаточно. Это ведь уже не рутина уже, а нечто связанное с креативностью. Компьютер можно кое-какие преобразования научить делать, но это после того, как они уже были кем-то придуманны. То есть одно дело, если я посто вижу, что это тривиальная задача уже решённая в прошлом Фурье полностью, то я применяю "ловкие" преобразования по инструкции и они меня приводят к заранее известному результату. А если это нетривиальная экзотика, то может у меня получится придумать, что с этим делать, а может и нет. Тут уже креативность получается. Для меня во всём школьном обучении эти преобразования были самой сложной и загадочной вещью, но я не побоялся поступить на математический факультет. В универе я уже ясно ощущаю, что сплошь в основном в упражнениях только классифицирую задачу и выполняю предписанные преобразования, чтобы решить задачу. И лешь редко нам дают что-то чуть экзотическое, где бы мы могли подумать и придумать что-то получается совсем редко и только что-то простое. Поэтому я уже перестал интересоваться практическими семинарами и "упражениями", а всё только учу лекционную теорию, где дают и преобразования и другие инструменты решения задач. Врочем это только тебе как школьнику интересно читать будет. Все это и так знают.
>> No.79647 Reply
>>79621
Ну и как тебе? Геометрия центряк? Вьюгин выбешивает? Жалко, что Буфетов больше вести не будет?
>> No.79652 Reply
>>79647
> Ну и как тебе?
Что "как"? Мне безразлично.
> центряк
Чтоу?
> выбешивает
Почему? Он обычный. Все обычные. Он тебе не нравится, потому что делает ошибки? Но все их делают. Например, у геометров тоже >>78994 с ошибкой.
> Буфетов
Ох, правда? Меня на последней лекции и последнем семинаре не было, но на следующие я буду обязательно совершать присутствие наверное. Впрочем, всё, что они говорят, есть в книжке, имя которой они сообщили с любезностью.
>> No.79656 Reply
Посоны, я первокур с фиизфака и в четверг у меня первая контрольная по матану. Нужен задачник с интегралами, пределами и разложением функций. Все на уровне основ.
>> No.79657 Reply
>>79656
Да ты охренел. Неужто про Демидовича никогда не слышал?
>> No.79661 Reply
>>79657
В Демидовиче нет типовых задач, на которых можно руку набить. Мне бы что-нибудь уровнем пониже, попроще.
>> No.79663 Reply
File: book.jpg
Jpg, 30.98 KB, 190×300 - Click the image to expand
edit Find source with google Find source with iqdb
book.jpg
>>79661
А рзаве есть куда ниже? Попробуй тогда это - Мордкович, Алгебра и начала анализа, 11 класс. Профильный уровень хоть осилишь?
>> No.79703 Reply
>>79624
В общем, можно разбираться в теории и быть неплохим математиком. Но в математическом творчестве это постоянно проделывается. В формальной математике же нет ничего, кроме выражений.
>> No.79707 Reply
>>79626
> В универе я уже ясно ощущаю, что сплошь в основном в упражнениях только классифицирую задачу и выполняю предписанные преобразования, чтобы решить задачу.
Ёлки, а мне казалось, что там нужно иметь ультрамозги и быть уберменшем, а ещё с утра до вечера доказывать теоремы.
>> No.79708 Reply
>>79707
Я сказал в упражнениях. В лекционной теории одни теореммы с доказательствами и без и определения идут.
>> No.79711 Reply
File: If-you-know-what-I-mean-mr-bean-meme.png
Png, 57.54 KB, 420×317 - Click the image to expand
edit Find source with google Find source with iqdb
If-you-know-what-I-mean-mr-bean-meme.png
>>79707
смотря какой универ же
>> No.79715 Reply
>>79711
Смотря какой курс же (в смысле, не год обучения, а цикл занятий).
>> No.79718 Reply
>>79708
А что именно за упражнения ты имеешь в виду?
>> No.79724 Reply
File: 0606-98.jpg
Jpg, 62.72 KB, 770×577 - Click the image to expand
edit Find source with google Find source with iqdb
0606-98.jpg
Аноны, как можно хорошо подготовиться к ЕГЭ по математике? Так, что бы без проблем решать задания C5 и C6? Какие хорошие книги школьного уровня можете посоветовать? Очень надо.
>> No.79734 Reply
File: Screenshot.png
Png, 52.66 KB, 619×213 - Click the image to expand
edit Find source with google Find source with iqdb
Screenshot.png
Аноны, ЧЯДНТ?
в) умножение натуральных чисел ассоциативно, но в ответе - нет
д) пример:
(0^2 + 1^2)^2 + 1^2 = 2,
0^2 + (1^2 + 1^2)^2 = 4
В ответе - да
ж) ну тут вообще пушка, если я не забыл со школы свойства степеней то чуть чуть порисовав пришел к ответу, что неассоциативно, однако в ответе да.

Чего я не понимаю в ассоциативности?
>> No.79736 Reply
File: Безымянный.JPG
Jpg, 16.52 KB, 806×153 - Click the image to expand
edit Find source with google Find source with iqdb
Безымянный.JPG
Анон, помоги научиться находить определитель по формуле на пикрелейтед. Я почти не понимаю, как эта конструкция превращается в сумму из n! слагаемых. Нам говорили, что на этой форме записано какое-то суммирование по индексам, но я ничего не понял. Прозреваю, что формула показывает, что надо брать элемент из разных строк и столбцов. А так как в одном слагаемом всегда есть индексы от 1 до n, то можно верхние индексы записать по порядку, а нижние образуют инверсию - это и есть то, что отличает слагаемые друг от друга. Объясните мне.
>> No.79737 Reply
File: 59346361.jpg
Jpg, 43.68 KB, 576×576 - Click the image to expand
edit Find source with google Find source with iqdb
59346361.jpg
>>79734
учебник проверяет, насколько ты уверен в своих решениях lol
>> No.79739 Reply
>>79734
С в) и д), я такой же как и ты. А на счёт "пушки" не согласен, у меня получилось доказать ассоциативность. Я сразу заменил эти муть в показателе на сигнум х.
>> No.79740 Reply
>>79734
в)
(x*y)*z = (2xy)z = 2(2xy)z = 4xyz
x*(y*z) = (2x)(yz) = 2(2x)(yz) = 4xyz
(x*y)*z = x*(y*z) 
Странно это.
>> No.79743 Reply
>>79734
У тебя правильные ответы сам пререпроверил на лимточки. Учебник - бака.
>> No.79744 Reply
>>79743
ж) - тоже чтоль?
>> No.79745 Reply
Черт >>79739 прав в д ответ да, я тоже бака.
x/|x| = sgn x x=/= 0
x#(y#z) = x * (y * z^(sgn(y))^sgn(x) = x * y^sgn(x) * z^sgn(xy) = (x * y^sgn(x)) z ^sgn(xy) = (x#y)#z
>>79743 кун
>> No.79746 Reply
>>79745
> в д ответ да
в ж конечно
и еще раз бака
>> No.79747 Reply
Спасибо, аноны, за отклик. Я только одного не понял вот чего.

(x#y)#z = (x#y)*z^(x#y)
(x#y) = [x*y^sgn(x)]
[xy^sgn(x)]#z = [xy^sgn(x)]z^[xy^sgn(x)]

Сопоставив с
>>79745
> (x * y^sgn(x)) z ^sgn(xy) = (x#y)#z
z^[x*y^sgn(x)] == z^sgn(xy) ??? Что-то оно мне не кажется верным.

Неужели я все-таки забыл школьную программу?
>> No.79755 Reply
>>79747
> (x#y)#z = (x#y)*z^(x#y)
сигнум перед второй скобкой потерялся.
> [xy^sgn(x)]#z = [xy^sgn(x)]z^[xy^sgn(x)]
То же самое.
>> No.79756 Reply
>>79747
Выделяй, пожалуйста, формулы как код (в справке написано, как это делать). Неудобно читать же.
>> No.79760 Reply
>>79755>>79756
okay
sgn[x*y^sgn(x)] = sgn(xy)
Да, немного неочевидно, но в конце концов я понел. Спасибо, матемач.
>> No.79764 Reply
>>79724
Завтра ищешь в Интернете книжку «Сборник задач для поступающих в ВТУЗЫ (С решениями).» Сканави, первую и вторую часть. Похуй если ничего не поймёшь. Затем выписываешь оттуда все формулы оттуда и заучиваешь. Потом зубришь, именно, сука, зубришь всю планиметрию и стереометрию в Евклидовом пространстве, а также действия с векторами в Декартовой системе координат. Когда сможешь решить из сборника любое задание из группы В, можешь идти дальше — тебя ждёт увлекательный мир «Все задания части С по математике» Панфёрова. С1, С2, С3, С4, С5, С6. Отсос олимпиадников / просто неудачников типа посещающих матшколы, которые решить ничего сложнее биквадратного уравнения по жизни не могут не заставит себя ждать и через полгода ты сможешь решать такие уравнения, что любой старый хуй будет принимать тебя в свой МГУ, Бауманку, ВШЭ, ИТМО или Физтех.
Алсо рекомендую изучить начала теории чисел и прочитать отсюда http://lj.rossia.org/community/studium/1717.html#cutid1 все три книги из категории «Начальный уровень.».
>> No.79769 Reply
File: for-dobrochan.jpg
Jpg, 132.22 KB, 2319×277 - Click the image to expand
edit Find source with google Find source with iqdb
for-dobrochan.jpg
Доброанон, помоги решить следующий номер:
Я нашел предельную функцию этой последовательности, она везде равно нулю.
Я нашел остаток ( модуль разности последовательности и предельной функции), он равен самой последовательности в условии. Далее по алгоритму проверки на равномерную сходимость нужно вычислить супремум этого остатка на данном нам множестве x, и вот тут то я не знаю, что делать. Пробовал считать производную и приравнивать ее к нулю, получается жуткое уравнению, которое не знаю, как решить. Вольфрамальфа выдает какое-то недорешение и не говорит, как решать.
>> No.79778 Reply
>>79769
Просто можешь оценить сверху arctg числом pi/2
>> No.79782 Reply
>>79778
Но ведь такие признаки есть для рядов, а не для последовательностей. Превратить последовательность в ряд можно разве что представив ее, как разность частичных сумм.
>> No.79785 Reply
>>79782
О каких признаках ты говоришь?
>> No.79786 Reply
>>79764
Предостерегу тебя от следования этому совету.
В последний год обучения упорол планиметрию по учебнику Атанасяна и в последние три месяца прочитал учебники Виленкина, Ивашева-Мусатова, Шварцбурда за 10-11 кл. (хотя я читал бо'льшую часть учебника за 10 кл. раньше, поэтому скорость получилась неплохая).
На задачи внимания почти не обращал несмотря на беспокойство наивных родителей. Решил многие по планиметрии из учебника и некоторые по алгебре и анализу из учебника, в основном более или менее интересные.
Прочитал немного ФЗФТШ про уравнения и неравенства, касающееся замен эквивалентными.
На ЕГЭ меня опьянила лёгкость заданий. За часть B набрал полный балл, а вот в C на максимальную оценку решил 1-е, 3-е и 4-е, а за оставшиеся набрал, кажется, 4 балла.
В итоге получил 87 баллов. Мне хватает с лишком.
>> No.79787 Reply
>>79785
З.Ы. Вот попробуй только списать, что я не понимаю тебя, потому что мне не достаёт интуиции, как вы в ответ на такие вопросы всегда говорите! Я спрашиваю не потому, что мне нужен ответ! А потому что я просто знаю, что нет никаких признаков, о которых ты говоришь, просто у тебя в голове каша и ты неу меешь мыслить логически!
>> No.79788 Reply
>>79787
Я за попкорном.
>> No.79789 Reply
>>79787
А о чем ты говорил, когда говорил, что можно оценить сверху arctg числом pi/2. Ну и что же это даст?
Я говорю а разных признаках сравнения рядов, когда например функциональный ряд мажорируют числовым рядом (признак вейерштрасса) например, или в более общем случае признак сравнения: ruwiki://Функциональный_ряд#.D0.9F.D1.80.D0.B8.D0.B7.D0.BD.D0.B0.D0.BA[...]D0.B8
>> No.79791 Reply
File: Math.png
Png, 4.43 KB, 279×97 - Click the image to expand
edit Find source with google Find source with iqdb
Math.png
>>79786
Когда даже не начинал подготовки — два с половиной месяца назад, то мог решить всё, кроме С6, С2 и С4. Баллов 80 точно бы набрал. А мне нужно больше.
Да и бугурт у меня от того, что я не могу решить пикрилэйтед — ведь дальше, в настоящей математике, будет сложней, наверное.
> На ЕГЭ меня опьянила лёгкость заданий.
Как в сборниках для подготовки или ещё легче?

%%>>79787
ахахахахахахаха)))))))))))))))))))))%%

Прошу прощения.
>> No.79794 Reply
>>79791
> в настоящей математике, будет сложней, наверное
Изучать осмысленные и мотивированные теории легче (как мне кажется), чем заставлять себя решать бессмысленные высосанные из пальца задачки.
>> No.79795 Reply
>>79789
Не надо зацикливаться на "признаках", |f_n(x)| <= pi / (2n)
>> No.79796 Reply
>>79791
> дальше, в настоящей математике,
В унике навряд ли будет что-то, касающееся настоящей математики. Или ты в Гарвард?
>> No.79797 Reply
>>79791
> Как в сборниках для подготовки или ещё легче?
Примерно так же. Я слабо в сборниках разбираюсь.
>> No.79798 Reply
>>79796
> Или ты в Гарвард?
Не, наверное в Бауманский или Физтех. Хочу побольше математики, но смежное. Какую-нибудь криптографию или разработку микропроцессоров.
P.S. Аноны, не знаете, как там с квантовыми компьютерами? Хоть один запилили, нет?
>> No.79799 Reply
>>79795
Оценил арктангенс числом пи/2. Проверил равномерную сходимость для нового ряда, ее нет, следовательно ничего не доказано. Если бы она была, то вероятно для исходного ряда тоже была бы.
>> No.79800 Reply
>> No.79801 Reply
>>79799
Почему нет? Вроде бы есть.
n|x| / (n^2 + x^2) <= 1/n
>> No.79803 Reply
>>79801
Куда ты x дел то? Мне надо не на обычную, а на равномерную сходимость проверить.
>> No.79804 Reply
>>79803
Я оценил сверху:
n|x| <= n (т.к. -1 <= x <= 1)
n^2 + x^2 >= n^2
Поэтому n|x| / (n^2 + x^2) <= n / n^2 = 1/n
Это и есть равномерная сходимость. Я доказал, что sup |fn(x)| <= pi/(2n). Следовательно fn(x) -> 0 равномерно.
>> No.79805 Reply
File: gibe_love.jpg
Jpg, 20.42 KB, 387×429 - Click the image to expand
edit Find source with google Find source with iqdb
gibe_love.jpg
>>79804
Кажется понял. То есть предельная функция - 0, а остаток получилось оценить сходящейся функцией => равномерно сходится. Спасибо тебе, анон.
>> No.79809 Reply
Аноны, мне иногда кажется, что изучение математики чем-то похоже на духовные практики. Все эти гипотезы легко примеряются на реальный мир, но, в отличие от всяких заёбов, конкретны, истинны и для них определены границы применимости. Все эти ряды, интегралы, графики вызывают у меня состояние, сходное с медитацией, что ли. А у вас такое бывает?
Упоротый-кун
>> No.79812 Reply
>>79809
"реальный мир" это фикция
Производные категории, симплектические многообразия, векторные расслоения и т.д. не менее реальны, а, вообще говоря, существенно более "реальны", чем, например, физическая абстракция, которая называется "электрон".
>> No.79813 Reply
>>79809
Не понял.
> Все эти гипотезы
> Все
lolwut? применять по-разному можно.
>> No.79840 Reply
File: поужинал-лейбницем.png
Png, 1.17 KB, 300×20 - Click the image to expand
edit Find source with google Find source with iqdb
поужинал-лейбницем.png
>> No.79850 Reply
>>79791
Расскажи решение, сам не могу придумать.
капча "будете послан", но я верю в твою доброту
>> No.79853 Reply
>>79812
Это глупость.
>> No.79859 Reply
>>79812
Никогда не волновался о реальности того, что относится к интересной мне математике. "Это же интересная игра, причём тут реальность?".
>> No.79864 Reply
Нужно за максимально короткий срок:
1)Научиться интегрировать.(Да, я мудак.)
2)Решать простые диффуры(от части не могу из-за пунтка 1).
3)Научиться решать все что связано с рядами хотя бы начало (суммы ряда с точностью, исследовать на сходимость и прочие ништяки).

1)Какую брать литературу с максимально внятными объяснением и примерами
2)Какой брать задачник в таком случае.
>> No.79869 Reply
>>79859
У каждого, кто изучает математику, имея цель заниматься этим профессионально, возникает вопрос об осмысленности математичесекой практики.
Думаю, немногие бы стали тратить время на игры.
Это я не к тому, что есть правильные и неправильные взгляды (хотя, по-моему, конечно, есть), а к тому, какая точка зрения является превалирующей.
>> No.79871 Reply
>>79812
> Производные категории
Какую книжку пользуешь?
>> No.79905 Reply
Котаны, я даже и не знаю, зачем я это тут пишу сейчас. Высказаться наверное, чтоб потеплело.
Пришел к нам чел, рассказал про группы, теорему Лагранжа, потом кольца-поля, что такое идеал, потом про конечные поля, и углубился в прикладистику в виде кода Хемминга и далее Рида-Соломона. Четыре пары лекций, четыре семинаров на это ушло. Где-то посреди всего сказал фразу - ну вообще, под то что мы с вами на двух занятиях пробижали где-нибудь на мехмате был бы семестрвый курс. Список литературы не выдал, и уж тем более никакой силлабус не написал.
Ну, короче теперь у меня БАЙКОНУР контрольная по пройденному. С аннотацией - "ну я еще не придумал что в контрольной будет. Но коншпектами своими можете пользоваться."
И вот я смеюсь как мудило сижу и не знаю чего теперь делать с ним таким, блядь.
>> No.79919 Reply
>>79905
это где?
>> No.79922 Reply
>>79813
> lolwut? применять по-разному можно
Мне, честно говоря, насрать на практическое применение. Всё, так или иначе, может найти своё применение в своё время.
Где-то читал про «теория чисел — основа современной криптографии, хотя сначала думали, что она бесполезна.».

Да и доставляет она мне, математика.
>> No.79926 Reply
>>79919
вшэ, примат (т.е. не матфак)
>> No.79940 Reply
>>79871
Гельфан-Манин & Weibel для начала, потом надо искать профильные рекомендации, например, на nlab'e
>> No.79973 Reply
>>79812
> а, вообще говоря, существенно более "реальны", чем, например, физическая абстракция, которая называется "электрон".
Распространённое заблуждение.
>> No.79975 Reply
>>79864
> Какую брать литературу с максимально внятными объяснением и примерами
Фихтенгольц альбо Зорич.
> Какой брать задачник в таком случае.
1)Лунгу, Письменный, Федин, Шевченко. "Сборник задач по высшей математике" в двух томах.
Издательство Айрис-пресс.
2)Волшебный кролик же.
>> No.80020 Reply
File: iepJiOP2tGo.jpg
Jpg, 31.20 KB, 373×322 - Click the image to expand
edit Find source with google Find source with iqdb
iepJiOP2tGo.jpg
Анон, поясни по-хардкору, в чём здесь надувательство.
>> No.80024 Reply
>>80020
Гипотенуза - на самом деле ломаная, вот и всё.
>> No.80025 Reply
>>80020
А самому подумать? Здесь и ребёнок догадался бы.
Снизу выпуклый четырёхугольник, сверху - впуклый.
>> No.80027 Reply
File: ddd.png
Png, 8.52 KB, 318×168 - Click the image to expand
edit Find source with google Find source with iqdb
ddd.png
>>80020
Может даже нет надувательства. Просто иллюстрация, насколько может быть значима невидимая в однмо случае погрешность или жирность зазоров. Для треугольника деление выглядело бы так.
>> No.80028 Reply
>>80027
Ответ уже 2 раза написали.
>> No.80030 Reply
>>80024
Спасибо! Каюсь, не заметил кривоту гипотенузы.
>> No.80031 Reply
>>80027
И тебе спасибо :3
>> No.80033 Reply
>>80025
Кстати да, я как-то просёк эту фишку, еще когда классе в девятом увидел. А потом забыл, погуглил и опять просёк.
>> No.80047 Reply
Нужно построить эскиз функции xx+3xy+yy+x=1. Я пытался выразить y, x, пытался подойти к ней параметрически (в т.ч. по аналогии с декартовым листом), но она не поддается. Анон, натолкни на дельную мысль.
>> No.80049 Reply
>>80047
А определить какими-то преобразованиями, что это за кривая никак нельзя? Вроде как можно как-то, но мы не проходили как а механики проходили.

мимопервокурсота
>> No.80050 Reply
File: MSP84081a3b9c4132296ih10000276fa22b1fee9704.gif
Gif, 4.13 KB, 300×298 - Click the image to expand
edit Find source with google Find source with iqdb
MSP84081a3b9c4132296ih10000276fa22b1fee9704.gif
>>80047 Держи: http://goo.gl/W0aty
Олсо, гугли "приведение квадратичной формы к каноническому виду".
>> No.80052 Reply
Матемач, предлагаю в следующем треде написать список литературы для школьников, готовящихся к ЕГЭ:
— Планиметрия и стереометрия. Киселёв. //Очень подробно разжёвана геометрия.
— Сборник задач для поступающих во втузы. Сканави. //Много задач.
— Справочник по элементарной математике. Выгодский. //Для того, чтобы вспомнить весь материал.
Сайт с задачами, который понравился мне больше всего http://www.reshuege.ru/
Пока так.
>> No.80053 Reply
>>80052
Предлагаю обсудить тему и по результатам написать список на pastebin.com.
>> No.80060 Reply
>>79864
> 1)Научиться интегрировать.(Да, я мудак.)
По Риману?
> 3)Научиться решать все что связано с рядами хотя бы начало (суммы ряда с точностью, исследовать на сходимость и прочие ништяки).
Числовые или функциональные?
>> No.80085 Reply
File: 114145_0.png
Png, 0.70 KB, 84×46 - Click the image to expand
edit Find source with google Find source with iqdb
114145_0.png
Как вычислить пикрелейтед?

(С этого экзамена: http://ium.mccme.ru/postscript/f06/anex1s_2.ps)
>> No.80104 Reply
>>80050
Я таки разобрался с этой хренью. Я, пытаясь построить эскиз, получил большое количество квадратных уравнений, где вспомогательный параметр был в коэффициентах. Сегодня знакомый меня носом туда ткнул: оставалось просто по стандартным формулам найти x и все получалось. Не очень красиво и просто конечно, но искомая гипербола вылезала.
Еще попробовал оси повернуть на 45 градусов для этой функции и тоже пришел к гиперболе. Почему 45 градусов - не спрашивайте - математическая интуиция =3
>> No.80301 Reply
>>79975
Что то смущает что под легкими вы дали зорича с фихтом. =\
>> No.80305 Reply
А подкиньте годной книженции по Банаховым алгебрам.
>> No.80309 Reply
Господа, задача:
lim(sum(k/(n^2+k^2)))
Сумма от 1 (кажется, но м.б. 0, но это, как видите, не существенно) до n, предел по n.
Задача не сложная, ответ выглядит примерно так (1/2)*ln(2)-1 или что-то вроде того. Т.е. нужно разглядеть здесь разложение логарифма в ряд Тейлора, но у меня пока не получилось.
>> No.80310 Reply
>>80309
> ответ выглядит примерно так (1/2)*ln(2)-1 или что-то вроде того
Еще, возможно, что ответ 0, но это вряд ли.
>> No.80316 Reply
>>80309
Да, ответ ln(2) / 2.
Я не знаю простого решения, есть стандартное для вычисления таких сумм, но сложное.
>> No.80319 Reply
>>80309
О, придумал простое решение: оцениваешь сумму снизу и сверху интегралом.
>> No.80321 Reply
Поцаны, как доказать иррациональность sin(pi/180)?
>> No.80322 Reply
Привет анончики. Вот у меня проблема такова: как доказать что множество всех конечных подмножеств счетного множества счетно?
>> No.80326 Reply
>>80322
http://www.diary.ru/~eek/p69004459.htm
см. комментарий Adjirranirr.
>> No.80327 Reply
>>80321
http://www.qbyte.org/puzzles/p070s.html

>>80322
Каждому подмножеству сопоставляем его двоичную запись
>> No.80329 Reply
>>80327
> Каждому подмножеству сопоставляем его двоичную запись
Не сказано, что множество числовое. См. >>80326
>> No.80338 Reply
>>80329
Вот по твоей же ссылке:
> Таким образом, каждому подмножеству натуральных чисел соответствует некоторая последовательность нулей и единиц. Для каждого конечного подмножества инвертируем соответствующую ему последовательность; тогда, она будет являться двоичной записью некоторого натурального числа. И, значит, каждому такому подмножеству соответствует единственное натуральное число, т.е. множество конечных подмножеств натуральных чисел счетно.
>> No.80381 Reply
>>80338
> подмножеству натуральных чисел
А если множество нечисловое, то его элементы предварительно нужно занумеровать. Лемма 2.
>> No.80393 Reply
>>80309
lim(sum(k/(n^2+k^2)))=lim(sum((k/n)/(1+(k/n)^2))/n)=int{0 to 1}(x/(1+x^2))dx=ln(2)/2
>> No.80397 Reply
>>80393
О, спасибо!
>> No.80404 Reply
А как это >>80085 решить?
>> No.80410 Reply
Анализ говно, вообще самый хуевый раздел математики, самый уродливый, некрасивый, неаккуратный, омерзительный. И читать по нему нечего, вообще все книги по нему говно, потому что везде есть этот пидорский интеграл Римана и доказательства с использованием эпсилона и дельты, они обе уже мне надоели. У меня почти ничего по анализу не получается. Уже неделю сижу, ничему не научился. Что делать не знаю.
>> No.80416 Reply
>>80410
> Анализ говно
> У меня почти ничего по анализу не получается.
Всё ясно.
> Уже неделю сижу, ничему не научился. Что делать не знаю.
Попробуй англоязычные книжки.
>> No.80424 Reply
Никто не посоветует хорошего учебника по C*-алгебрам?
>> No.80426 Reply
Поцоны, накидайте, пожалуйста, годных книжек по теории групп.
>> No.80427 Reply
File: zaporozhetc.jpg
Jpg, 32.28 KB, 200×308
edit Find source with google Find source with iqdb
zaporozhetc.jpg
File: 9781409269519.jpg
Jpg, 14.56 KB, 189×280
edit Find source with google Find source with iqdb
9781409269519.jpg

Пикрелейтеды наглядно показывают, насколько западная литература превосходит местную.
>> No.80428 Reply
File: 1350312683032.png
Png, 0.97 KB, 300×20 - Click the image to expand
edit Find source with google Find source with iqdb
1350312683032.png
Анон, помоги тупой первокурсоте въехать в матрицы, поделись мануалаи для идиотов. Дошли до главы "системы линейных уравнений" по Комиссаровой, а до сих пор отстаю.
>> No.80430 Reply
>>80424
В http://lj.rossia.org/community/studium/1717.html#cutid1 смотрел?

>>80426
Тебе туда же. Там "2. В.Б.Алексеев. Теорема Абеля в задачах и решениях" (оче клёвая) и всё что в группе "Алгебра" (наверное. Сам я их не читал).
>> No.80431 Reply
>>80410
Хорошая паста. Схоронил.
>> No.80432 Reply
А я тоже спрошу. Подскажите годных книжек по статистике? Кроме очевидных Ивченко-Медведева.
>> No.80438 Reply
File: Вавилов-Н.-Конкретная-теория-групп.pdf
Pdf, 1.67 KB, 595×842 - Click the image to get file
Вавилов-Н.-Конкретная-теория-групп.pdf
>>80426
качай сам @ с другими делись
>> No.80454 Reply
File: MSP37741a3d02eca8eh83f7000049cbb9672fhhe71b.gif
Gif, 0.91 KB, 75×51 - Click the image to expand
edit Find source with google Find source with iqdb
MSP37741a3d02eca8eh83f7000049cbb9672fhhe71b.gif
>>80404 Представляешь в виде функционального ряда, интегрируешь по x, сумму логарифмов расписываешь через логарифм произведения, у полученного произведения легко найти результат для любого конечного числа членов и перейти к пределу. Потом осталось продифференцировать результат и вычислить его в точке x=1.
>> No.80458 Reply
>>80438
Откуда взял?
Книжка очень годнаяно автор очень любит себя.
>> No.80470 Reply
>>80432
Присоединяюсь. Видел томик Колмогорова про тервер и статистику(?), но мне почему-то казалось, что там только тервер... (качал когда-то)
>> No.80480 Reply
>>80454
Спасибо. Ещё можно воспользоваться тригонометрическим тождеством
tg(x) = ctg(x) - 2ctg(2x)

Тогда все члены, кроме крайних, сокращаются.
Частичные суммы равны: ctg(1/2^n) / 2^n - ctg(1) -> 1 - ctg(1)
>> No.80510 Reply
>>80470

Я уже постил список:
>>76543

К нему добавлю вроде бы годноту (сейчас читаю):
Keener - Theoretical Statistics: Topics for a Core Course
Barber - Bayesian Reasoning and Machine Learning
>> No.80671 Reply
>>80510
Спасибо, няш!
>> No.80703 Reply
Анончики, что можно сказать про счетность множества всех алгебраических многочленов с рациональными коэффициентами. И как доказать что оно счетно/несчетно.
>> No.80705 Reply
>>80703
Множество всех конечных последовательностей натуральных чисел счетно.
>> No.80711 Reply
Почему множество всех последовательностей из нулей и единиц несчетно?
Если выписать эти последовательности так.
000000...
100000...
010000...
110000...
...
Каждой можно дать число, являющееся представлением его в двоичной как написано. Разве нет?
>> No.80712 Reply
>>80711
Это множество последовательностей, заполненных нулями с какой-то позиции. Например, последовательности 10101010... там нет.
>> No.80716 Reply
>>80711
Нет. Будут записи с бесконечным количеством единиц 010101010101....
>> No.80725 Reply
>>80711
> Каждой можно дать число, являющееся представлением его в двоичной как написано
От противного. Предположим, что дали, что каждой последовательности нулей и единиц сопоставлено натуральное число. Упорядочим последовательности нулей и единиц по возрастанию сопоставленных натуральных чисел. Построим теперь вот такую последовательность нулей и единиц: из i-го числа возьмём i-й символ, инвертируем его и выпишем. То есть, например, для
[0]00000...
1[0]0000...
01[0]000...
110[0]00...
...
выписанная последовательность нулей и единиц: 1111...
По предположению, этой последовательности соответствует натуральное число K. Однако по построению от последовательности, стоящей на K-м месте, наша последовательность отличается K-м символом. Имеем противоречие.
>> No.80727 Reply
>>80725
Кстати, аноны, почему множество действительных чисел и множество всевозможных последовательностей из нулей и единиц равномощны?
>> No.80728 Reply
>>80727
Каждое действительное число из отрезка [0;1] - двоичная последовательность (двоичная дробь) за исключением счетного множества.
>> No.80730 Reply
>>80727
|R| = |2^N| а любое подмножество N задается бесконечной последовательностью 0 и 1.
>> No.80731 Reply
>>80728
> Каждое действительное число из отрезка [0;1] - двоичная последовательность (двоичная дробь)
Почему?
>> No.80732 Reply
>>80731
В одну сторону (последовательность - число):
Потому что последовательность частичных сумм ряда a0+a1/2+a2/4+... образует фундаментальную последовательность, и два предела отличаются, если у них разные ai, но не таким образом: a0+a1/2+...+(ai+1)/2^-i+0/2^-(i+1)+... и a0+a1/2+...+a1/2^-i+1/2^-(i+1)+1/2^-(i+2)+...
В другую: если делить отрезок пополам, то заданное число будет либо больше половины, либо меньше, либо равно. Выберем ту половину, внутри которой или на левой стороне которой лежит число, если она левая - запишем 0, если правая 1., потом разделим её и т.д. Последовательность сходится к одной точке.
>> No.80734 Reply
>>80732
> и два предела отличаются, если у них разные ai, но не таким образом
Пардон, не понял эту фразу.
>> No.80735 Reply
>>80734
Две суммы таких рядов различаются, если у них вид не a0+a1/2+...+(ai+1)/2^-i+0/2^-(i+1)+... и a0+a1/2+...+a1/2^-i+1/2^-(i+1)+1/2^-(i+2)+... (a0,a1,... = 0 или 1) и если у них есть несовпадающий член.
Таким образом, континуум последовательностей соответствует континууму чисел (их пределов) в [0;1].
>> No.80736 Reply
File: дурачок-саи.png
Png, 0.95 KB, 300×20
edit Find source with google Find source with iqdb
дурачок-саи.png
File: 60541 green_eyes ...
Jpg, 428.44 KB, 1433×1012
Your censorship settings forbid this file.
unrated

Помогите решить задачу, я с этой вышкой не в ладах. Для лучшего восприятия представим, что у меня есть 684 разных фигурок Сайи и для полного счастья мне надо достать ещё 321, так как фулл сет равен 1005 фигуркам. Каждый день я жру в Макдональдс и я рандомом получаю одну из 1005 фигурок. Из-за этого я жирный боров, 3 центнера весом, отказывает печень и сердце, но я отец, папка апокалипсиса и буду идти до конца. В общем, какова вероятность достать их все, прежде чем я умру от ожирения?
>> No.80737 Reply
>>80736
0/10
>> No.80738 Reply
>>80736
В среднем тебе придётся собирать ещё 17.5 лет
>> No.80741 Reply
>>80737
>>80738
Это не ответ.
>> No.80742 Reply
>>80736
> В общем, какова вероятность достать их все
Меньше единицы. Если бы ты дал больше информации, можно было бы сравнить вероятность получения всех фигурок и вероятность смерти от ожирения.
>> No.80778 Reply
>>80731
Просто запиши число в двоичной системе счисления.
>> No.80781 Reply
>>80778
> Просто запиши число в двоичной системе счисления.
Нет, нельзя.
>> No.80785 Reply
>>80781
Почему?
>> No.80787 Reply
>>80785
При диагонализации может получиться последовательность, которая не будет являться двоичной записью десятичного числа. Нужно искать более хитрые способы установить взаимно-однозначное соответствие.
>> No.80789 Reply
>>80787
Нет, любая последовательность из нулей и единиц является записью двоичного числа.
Или ты работаешь в терминологии, что 0.0(1) - не запись двоичного числа, так как этому числу соответствует ещё и другая запись 0.1?
>> No.80812 Reply
А я изучаю "Студенческие чтения МК НМУ" и заочно наслаждаюсь барочной атмосферой НМУ 90-х годов.

Тихая размеренная жизнь в паутине арбатских переулков, второкурсники (все шесть человек) собрались в пустой аудитории и решают листочки по топологии, бородатые фрики в коридоре обсуждают недавно доказанные гипотезы о мотивных когомологиях, по лестнице в конференц-зал поднимается Арнольд, сегодня он читает лекцию о группах Ли, а на следующей неделе обещал заехать Манин и рассказать что-то интересное об алгебраических кривых.
>> No.80816 Reply
>> No.80819 Reply
>>80816
да-да, после этой душевной пасты я и стал name-ёбом.

Вообще, я и сам из быдло-среды, поэтому чувства того товарища мне понятны.
Когда впервые оказываешься в подобной тусовке и видишь своего ровесника - нерда с жидкими усиками и в вельветовых штанах и сандалях, который в то же время знает математику вдесятеро лучше тебя (т.е. является более респектабельной персоной по критериям данной социальной группы в данных условиях), то испытываешь, осознанно или нет, сильный butthurt, ведь это он выглядит жалко и разговаривает писклявым голосом, а хуй, получается, не он, а ты.

Главное, перетерпеть этот момент и разобраться, что к чему. Потом оказывается, что и фрики не так умны, и ты не такой тупой, и наука не такая уж сложная, а отрицательные качества, которые ты им приписывал являются отчасти твоей собственной проекцией. А потом оказывается, что и в общении эти фрики с жидкими усами оказываются интереснее многих твоих знакомых. К тому же их не так много, например, среди тех моих знакомых, которые действительно хорошо разбираются в математике для студенческого уровня, нет никого, кто бы был несчастно одинок или существенно непопулярен среди своих знакомых-нематематиков.
>> No.80826 Reply
>>80816
Но >>76132 же о младших курсах МГУ, я так понял.
>> No.80827 Reply
>>80826
Не, малый мехмат немного другая тусовка. Те, кто поступает в матшколы, обычно туда ходить перестают, если раньше ходили.

Еще замечу, что
> сразу повеяло запахом Арнольда
   - это не просто фигура речи.
>> No.80829 Reply
>>80819
Если ты пришел в эту тусовку в возрасте >18 лет, тебя эта паста мало касается. Это про матшкольную среду.
>> No.80832 Reply
http://dobrochan.ru/u/res/74216.xhtml#i80773
Перепостить лень. Если поможете - буду премного благодарен.
>> No.80838 Reply
>>80832
Смеешься что ли? Это (первый пример) - парадокс Банаха-Остроградского в классической формулировке. Предел неопределен. Уже много после оказалось, что его удобно рассматривать как случайную величину, что дало начало целой главе стохастического анализа, но это не школа и даже не вузовская программа.
>> No.80840 Reply
File: поехавший.jpg
Jpg, 332.82 KB, 811×941 - Click the image to expand
edit Find source with google Find source with iqdb
поехавший.jpg
>>80838
Причём тут танки няша?
>> No.80860 Reply
> Начальный уровень:
> 1. Р.Курант, Г.Роббинс. Что такое математика?
> 2. В.Б.Алексеев. Теорема Абеля в задачах и решениях
> 3. Я. Зельдович, И. Яглом: Высшая математика для начинающих физиков и техников
Ссылки посмотрел.
Все не то.
Нужен какой-нибудь один супер-учебник по математике. От основ (раскрывания скобочек и сложения дробей) до не знаю чего (до уровня 1-2 курса может быть). Например, по химии аналог знаю - "Начала химии" Кузьменко.
>> No.80863 Reply
Аноны, поясните по-хардкору где тут ошибка.

x = 0.(9)
10x = 9.(9)
10x = 9 + x
9x = 9
x = 1
1 = 0.(9)
>> No.80867 Reply
File: 1317995028053.jpg
Jpg, 23.98 KB, 316×341 - Click the image to expand
edit Find source with google Find source with iqdb
1317995028053.jpg
>> No.80869 Reply
>>80863
Здесь нет ошибки.
>> No.80872 Reply
>>80860
Для чего тебе нужна математика. От этого будут крайне отличаться рекомендации.
  
Для справки - первые 2 курса инженерных специальностей это:
пределы/производные/интегралы/функции многих переменных
матрицы и ленейные отображения
аналитическая геометрия
может еще что-то. В один учебник никак не влезет да и сильно разные разделы математики это.
на математических специальностях этот список будет больше в раза в 3
Школьная программа сама по себе не нужна.
>> No.80879 Reply
>>80869
Там, ровно как и на википедии, сплошь ошибки и отсутствие вменяемой аргументации. Там даже "пруфы" врут - не поленился, проверил, в том же Рудине такого долбоебизма нет (как и в любом нормальном учебнике).
Миф уровня форумов, борд и желтой прессы.
Про ошибки - srsly, not this shit again.
>> No.80884 Reply
>>80860
Как правило учебники "сразу обо всём" намного хуже, чем учебники о чём-то конкретном.
Посмотри Ильин, Куркина "Высшая математика".
Это отвратительно, но может быть тебе подойдёт :3
>> No.80885 Reply
>>80872
Мне нужно сдать, главное, экзамен (ЕГЭ) по математике. Сдать хорошо. Школьный курс я давно уже забыл, но вспомню. Но не только ради экзамена математика нужна. Поступать буду на химика, так что математика - не первое, но и не последнее дело. Знать надо. Знать не на отъебись, а нормально. Светилом матанализа быть не собираюсь, но и профаном оставаться больше не могу. Надеюсь, понятно объяснил.
>> No.80889 Reply
>>80885
Но те, кто хорошо сдаёт ЕГЭ и добивается успеха в университетской математике — профаны в математике.
Алсо, в математике столько всего. Есть интуиционистская математика, есть конструктивистская, есть формалистическая математика и бурбакизм есть реалистический платонизм. И в каждой из них много всего.
Если считаешь математику наукой, то наверняка обойдёшься без представлений о применяемой логике и согласишься с интуитивным представлением логичности. Дальше ты, вероятно, выберешь реализм. Потом тебе надо определиться с уровнем материала. Потом примерно определяешься с разделом. Потом выбираешь литературу.
Если надо, то наобум по низкому уровню научной математики подскажу Ван Дер Вардена, но для ЕГЭ это не то, конечно.
Но для ЕГЭ это не то. Тебе нужны учебники средней школы.
>> No.80891 Reply
>>80860
Выгодский. Справочник по элементарной математике.
+
Выгодский. Справочник по высшей математике.
Я серьёзно. (Там, правда, упражнений маловато, насколько я помню - вот тут помочь не могу. Разве что какого-нибудь Ткачука попробуй, но он слишком сложный, кажется)
>>80879
Что именно на вики смотрел? Что за пруфы, которые врут?
>> No.80892 Reply
>>80889
> Но те, кто хорошо сдаёт ЕГЭ и добивается успеха в университетской математике — профаны в математике.
А те, кто сдают плохо, либо те, кто сдают хорошо, но не добиваются успеха в универской математики - сплошь гении, ага.

И вообще, ты какой-то странный, бро.
>>80885
> Мне нужно сдать, главное, экзамен (ЕГЭ) по математике.
Таки Ткачук. Математика — абитуриенту. Есть ещё кто-то типа него, но другой, но его я не помню.
>> No.80893 Reply
>>80892
> А те, кто сдают плохо, либо те, кто сдают хорошо, но не добиваются успеха в универской математики - сплошь гении, ага
Mmmaximum demagog. Но я отвечу. Имелось в виду, что большинство. Видишь ли,
> Алсо, в математике столько всего. Есть интуиционистская математика, есть конструктивистская, есть формалистическая математика и бурбакизм есть реалистический платонизм.
В большинстве университетов, насколько я понимаю, платонизм и интуитивное представление о логике, профанство в классической математике. Об интуиционистских и конструктивистских концепциях, скорее всего, редкие люди знают, а большинство как бы знающих плохо могут, и их интуиционизм или конструктивизм на низком уровне. Слово формализм, поди, популярнее, но наверняка тоже убог.
>> No.80894 Reply
>>80891
> Что именно на вики смотрел? Что за пруфы, которые врут?
Все, и на английской, и на русской. Объяснять долго, да и лень. Можешь сам копнуть отсылки?
>> No.80896 Reply
>>80863
Действительное число - это по определению класс эквивалентности сходящихся последовательностей рациональных чисел.
0.(9) := {0.9, 0.99, 0.999, 0.9999, ... }
1 := {1/1, 1/1, 1/1, 1/1, 1/1, ... } (слева 1 действительное число, справа рациональное).
0.(9) и 1 сходятся, эквивалентны, значит, задают одно и то же число.
>> No.80899 Reply
File: flaviy_magmar.jpg
Jpg, 41.27 KB, 190×171 - Click the image to expand
edit Find source with google Find source with iqdb
flaviy_magmar.jpg
>>80863
Это не ошибка и не парадокс, это демонстрация баги в твоём мышлении. Ты мыслишь действительное число как одну неделимую сущность, а оно на самом деле сложное, у него "под капотом" находится очень много всего.
>> No.80902 Reply
>>80885
> так что математика - не первое, но и не последнее дело.
> Светилом матанализа быть не собираюсь, но и профаном оставаться больше не могу.
Попробуй в добавок к посоветованному Д.Кнут "Конкретная математика". Полезнейшая книжка, из нее можно подчерпнуть громадной колличество способов решения разных задач, и просто полезных фактов о которых обычно не говорят в школе/вузе. Есть куча примеров и упражнений. Написана простым языком с некоторым количеством юмора.
>> No.80903 Reply
>>80902
Я где-то видел мнение, что это всё показуха, популизм, дешёвый пиар, псевдонаука, и всё такое. Брешут?
>> No.80904 Reply
Аноны, а расскажите после чего вам стало интересно заниматься математикой, после какой книги например или, может, задачи. Или почему вам интересно ей заниматься?
>> No.80905 Reply
>>80904
Ещё с начальной школы понравилось крутить-вертеть цифры и буквы. В средней школе посмотрел на теорему косинусов, сравнил её с формулой квадрата разности, сказал "ба!" и всё заверте...
>> No.80921 Reply
File: 1235247832137.jpg
Jpg, 34.77 KB, 517×373
Your censorship settings forbid this file.
unrated
>> No.80922 Reply
>>80921
Щито эх?
>> No.80926 Reply
>>80922
Неправильно, вот щито. Но переубеждать не буду, тысячи раз перетирали.
>> No.80927 Reply
>>80926
Нет, анон, так просто я тебя не отпущу. Раз уж начал разговор, то будь любезен переубедить или дать ссылочку на пост, где описано, почему неправильно.
>> No.80928 Reply
Опять он единицу принес.
Аноны, кстати, "Теоретическая арифметика" Арнольда - рекомендуете ознакомиться?
>> No.80931 Reply
>>80928
это не тот Арнольд
посмотрел содержание, в книжке много полезных вещей, но много и дремучей архаики, читать 400 страниц, мне кажется, не стоит

А на счёт числа 0.(9) всё предельно ясно объяснил вот этот ==>>80896 товарищ
иногда полезно знать определение, лол
>> No.80937 Reply
>>80926
Все там правильно, лолка. У десятичного числа в такой записи две формы представления.
>> No.80946 Reply
>>80896
> 0.(9) := {0.9, 0.99, 0.999, 0.9999, ...}
Это неверно.
> класс эквивалентности сходящихся последовательностей
> 0.(9) и 1 сходятся, эквивалентны, значит, задают одно и то же число.
Отношение эквивалентности говорит лишь о том, что 0.(9) ~ 1, а не "задают одно и то же число".
>> No.80948 Reply
>>80946
Число - это и есть класс эквивалентности.
0.(9) и 1.0 - это просто разные способы обозначить один и тот же класс (число).

Вообще, я не понимаю, почему так тяжело принять факт, что 0.(9) - это просто другая запись 1.0? По-моему это очевидно.
>> No.80952 Reply
>>80896
>>80948
только там присутствует опечатка
> Действительное число - это по определению класс эквивалентности сходящихся последовательностей рациональных чисел.
а должно быть "класс эквивалентности фундаментальных последовательностей"
в этом как бы и суть
>> No.80953 Reply
>>80948
> 0.(9) и 1.0 - это просто разные способы обозначить один и тот же класс (число).
Ещё раз, нет такого.
А ещё вот это обоснуй, пожалуйста:
> > 0.(9) := {0.9, 0.99, 0.999, 0.9999, ...}
> Вообще, я не понимаю, почему так тяжело принять факт, что 0.(9) - это просто другая запись 1.0? По-моему это очевидно.
Это не очевидно, такого в математике, к счастью, вообще нет. Как и инфинитизималей/бесконечностей в R. Так, например, 0.(9) в R - это всего-навсего десятичная запись с количеством 9 ---> бесконечности, т.е. количество очень велико и приближено к бесконечности.
С таким же успехом ты мог бы задать число через (0.1)^n, n ---> бесконечности и приравнять к нулю, хотя сама функция (0.1)^n никогда нуля и не достигнет.
>> No.80954 Reply
>>80953
> > 0.(9) := {0.9, 0.99, 0.999, 0.9999, ...}
Не то процитировал, ну да ладно.
>> No.80955 Reply
>>80946
> Это неверно.
Было бы интересно взглянуть на твоё определение 0.(9). Покажешь?
> Отношение эквивалентности говорит лишь о том, что 0.(9) ~ 1, а не "задают одно и то же число".
По определению, вещественное число - это множество последовательностей.
Последовательности {a_n} и {b_n} называются эквивалентными, если
lim n->~ (a_n - b_n) = 0.
У тебя иное определение действительного числа? Какое?

>>80952
Последовательность является сходящейся тогда и только тогда, когда фундаментальна. Критерий Коши же.
>> No.80956 Reply
>>80953
У вас в голове каша, вам нужно почитать, что такое пополнение по норме/метрике.
enwiki://Completeness_(topology)#Completion
enwiki://Construction_of_the_real_numbers#Construction_from_Cauchy_sequences
>> No.80957 Reply
>>80955
> Последовательность является сходящейся тогда и только тогда, когда фундаментальна
Это в полных пространствах, пространства по определению полны, если такое выполняется.
При построении действительных чисел у нас не имеется ничего, кроме рациональных и из них нужно изготовить действительные. Это возможно только потому, что рациональные числа не являются полным пространством, т.е. некоторые фундаментальные последовательности не сходятся и мы можем перейти к пополнению, элементами которого и являются классы эквивалентности фундаментальных.
>> No.80958 Reply
>>80957
Хм. Спасибо за уточнение.
>> No.80959 Reply
>>80903
Не знаю о чем ты. Книга очень "конкретна". Не содержит в себе того что можно назвать "псевдонаукой". По словам авторов цель книги научить читателя не абстрактной математике, а более реальной практической. С чем книга с лихвой справляется. И почему я ее и посоветовал.
> показуха, популизм, дешёвый пиар
Кнут(создатель TeX, автор искусства программирования) и без того очень известная личность, чтобы нуждаться хоть в каком-то пиаре.
>> No.80960 Reply
>>80958
пространство изометрично вкладывается в своё пополнение, поэтому, будучи погружёнными в действительные числа, фундаментальная последовательность рациональных конечно сходится, как раз к тому элементу, который она представляет. Это как бы тавтология.
>> No.80962 Reply
Следующий тред: >>80961
>> No.80963 Reply
>>80953
> Так, например, 0.(9) в R - это всего-навсего десятичная запись с количеством 9 ---> бесконечности
Да нет же, числа существуют отдельно от своих записей.

Любое действительное число - это какой-то класс эквивалентности последовательностей рациональных. Чтобы сложить 2 действительных числа, нужно взять любые последовательности {an}, {bn} из соответствующих классов и построить последовательность {an + bn}. Можно доказать, что она тоже будет фундаментальной, и её класс эквивалентности не зависит от выбора конкретных {an}, {bn}

Десятичные дроби - это одна из форм записи действительных чисел. Мы сами придумываем правила этой записи и проверяем, что нет противоречий со свойствами действительных чисел. В рамках этой формы записи 1.0 = 0.(9), т.к. оба числа находятся в одном классе эквивалентности (хотя их запись отличается, но это недостаток выбранной нами формы; для устранения неоднозначности можно, например, запретить бесконечный хвост из девяток, но это не обязательно)
>> No.80964 Reply
>>80953
Вот, кстати, почему не возникает вопросов к тому, что 2/4 = 1/2? Тоже ведь разные записи одного числа.
>> No.80970 Reply
>>80964
> Да нет же, числа существуют отдельно от своих записей.
Что "нет", если 0.(9) - это 0.999... - где ... значит количество девяток {n--->бесконечности}. Вот и всё.
>>80964
Это ты вообще к чему?
>> No.80973 Reply
>>80970
Няш, ты понимаешь, что есть три различных сущности:
1) действительное число - множество эквивалентных последовательностей
2) предел (рациональное или иррациональное число), к которому сходится любая из последовательностей из действительного числа
3) символ, которым обозначается некое действительное число
?
>> No.80977 Reply
>>80973
Да. Надеюсь, и ты разграничиваешь последовательности и их пределы.
>> No.80986 Reply
Привет, матемач.
Можно ли нечисленно решить уравнение x + e^(-x) = 0 ?
>> No.80991 Reply
>>80986
Уравнение эквивалентно x * e^(x) = -1
У него бесконечно много решений, все комплексные

См. enwiki://Lambert_W_function

Что значит "нечисленно решить"?
На эту тему есть статья, которая предлагает точное определение: http://math.mit.edu/~tchow/closedform.pdf
Насколько я понимаю, пока ответа нет. Есть недоказанные гипотезы.

Перекатывайся в новый тред: >>80961
>> No.81002 Reply
>>80736
Да блин, что так трудно помочь? Можно и по другому поставить задачу: У мастера есть 684 детали для механизма из 1005, т.е. для полного комплекта не хватает ещё 321 деталь. Есть мешок, в котором содержатся все детали в бесконечном количестве и их выпадение абсолютно рандомно. Какова вероятность достать все недостающие детали?
Я могу сказать точно, что 685-ю деталь он достанет с вероятностью в 32% и последнюю с вероятностью в 0,0(1)%, а полностью как вся задача с решением выглядеть будет?
>> No.81003 Reply
>>81002
Ты слишком нервный няша
В прошлом условии был какой-то срок, к которому нужно успеть ("прежде чем я умру от ожирения"). От него зависит ответ.
Если каждую деталь нужно достать с первого раза, то результат - произведение вероятностей.

321/1005 * 320/1005 * ... * 1 / 1005 = 321! / 1005^321

И перекатывайся в новый тред: >>80961
>> No.81006 Reply
>>80991
Спасибо за разъяснения, няша, это ровно то, что мне и было нужно.
Одно только не совсем понимаю - там часом не e^(-x) в эквивалентном будет?
>> No.81091 Reply
>>81003
Ок, спасибо.
>> No.113625 Reply
File: gelfand-shen-algebra.pdf
Pdf, 0.74 KB, 397×567 - Click the image to get file
gelfand-shen-algebra.pdf


Password:

[ /tv/ /rf/ /vg/ /a/ /b/ /u/ /bo/ /fur/ /to/ /dt/ /cp/ /oe/ /bg/ /ve/ /r/ /mad/ /d/ /mu/ /cr/ /di/ /sw/ /hr/ /wh/ /lor/ /s/ /hau/ /slow/ /gf/ /vn/ /w/ /ma/ /azu/ /wn/ ] [ Main | Settings | Bookmarks | Music Player ]